0% found this document useful (0 votes)
450 views53 pages

Rheumatoid Arthritis Case Studies

A 40-year-old woman presents with joint pain and swelling, morning stiffness, and x-ray findings of osteopenia and bone erosions. Anti-CCP antibody testing would have the greatest specificity for diagnosing her condition. A patient with longstanding RA on medications calls with new knee pain, swelling and difficulty bending; aspirating the joint is the next best step to rule out infection. A RA patient on methotrexate presents with eye pain, redness and sensitivity; they are most likely experiencing scleritis, an eye complication seen in active RA. Subcutaneous nodules seen in a RA patient on the elbows would show pallisading histiocytes on biopsy.

Uploaded by

amere
Copyright
© © All Rights Reserved
We take content rights seriously. If you suspect this is your content, claim it here.
Available Formats
Download as PDF, TXT or read online on Scribd
0% found this document useful (0 votes)
450 views53 pages

Rheumatoid Arthritis Case Studies

A 40-year-old woman presents with joint pain and swelling, morning stiffness, and x-ray findings of osteopenia and bone erosions. Anti-CCP antibody testing would have the greatest specificity for diagnosing her condition. A patient with longstanding RA on medications calls with new knee pain, swelling and difficulty bending; aspirating the joint is the next best step to rule out infection. A RA patient on methotrexate presents with eye pain, redness and sensitivity; they are most likely experiencing scleritis, an eye complication seen in active RA. Subcutaneous nodules seen in a RA patient on the elbows would show pallisading histiocytes on biopsy.

Uploaded by

amere
Copyright
© © All Rights Reserved
We take content rights seriously. If you suspect this is your content, claim it here.
Available Formats
Download as PDF, TXT or read online on Scribd
You are on page 1/ 53

ACR

LEARNING
1. A 40-year woman comes in with 4 months of worsening joint pain and swelling of the small
joints of the hands, wrists, feet and ankles, severe fatigue, and 2 hours of morning stiffness which
improves as the day goes on and with movement. You obtain an x-ray of the hands and wrists
that demonstrates diffuse osteopenia and marginal erosions in the MCPs and carpal bones of the
wrist.

The test which has the greatest specificity for this patient’s diagnosis is:

A. Rheumatoid factor (RF)


B. HLA-B27
C. Anti-CCP antibody (CCP)
D. ANA
E. Double stranded DNA (dsDNA)

2. A 50 year old with a long history of Rheumatoid Arthritis on methotrexate and etanercept calls
to report 3 days of gradually worsening knee pain with swelling, difficulty bending the knee and
trouble weight bearing. The knee appears red. All of the other joints that have been previously
active are asymptomatic.

The next best management for this patient is:

A. Plain films of the knee


B. Aspirate the joint
C. Start high dose steroids
D. Increase the methotrexate dose
E. MRI of the knee

3. A 35 year old with RA currently treated with methotrexate alone, previously with good
control, comes in to see you with new and painful red right eye. On exam the eye is read with
limbal sparing. The patient is sensitive to light and pressure on the globe produces pain.

Which of the following eye complications would most likely be seen in a patient with active
Rheumatoid Arthritis?

A. cataract
B. scleritis
C. uveitis
D. conjunctivitis

4. You are seeing a 60-year-old patient with longstanding erosive RA, on methotrexate with a
positive RF and CCP who comes in to see you because he noted several swellings on the
extensor surface of the elbow. On exam, these appear to be flesh colored and are moveable, with
a rubbery feel. They are nontender.

The histology which best represents the pathophysiology of these nodule is:
A. pallisading histiocytes
B. septal panniculuitis
C. neutrophilic infiltrate
D. Negatively birefringent crystals
E. Prior to undergoing surgery in which general anesthesia is required all RA patients
should undergo the following testing:
F. C-spine x-ray
G. Lumbar x-ray
H. Pulmonary function testing
I. Lower extremity dopplers

1) The correct answer is C.

CCP antibody is the most sensitive test for the diagnosis of RA with a sensitivity and specificity
of greater than 98%. RF is much less specific and only about 60% sensitive. ANAs can be seen
in RA in addition to lupus and other connective tissue disease so is not specific for the diagnosis.
HLAB27 can be seen in association with seronegative arthritis. Double stranded DNA is specific
for lupus and is not seen in RA.

2) The correct answer is B.

The next best step is to aspirate the joint to rule out infection, particularly since there is one joint
out of proportion to the rest. Patients on immunosuppressive therapy sometimes fail to present
with the common signs of infection (i.e., fever) and suspicion as such should be high. Aspiration
is the only way to diagnose infection in this patient. X-rays and MRI may show an effusion but
cannot definitively diagnose infection. Increasing methotrexate or adding steroids until infection
is ruled out would be wrong.

3) The correct answer is B.

This patient has scleritis, which can be seen in up to 1% of RA patients. Uveitis is a more
common problem in patients with seronegative arthritis such as spondyloarthritis or psoriatic
arthritis. A cataract can be seen in patients with RA, particularly as a result of treatment with
steroids, however it is not commonly due to the disease itself. Conjunctivitis is an infectious
complication of the eye and would be due to immunosuppression, not the underlying disease.

4) The correct answer is A.

These are subcutaneous nodules, more common in more severe RA patients who, like this patient
are seropositive for RF and/or CCP antibodies. Nodules may be a sign of more active disease.
They can occur in 20-35% of patients. The most common location is on the extensor surfaces of
the elbows. Methotrexate is thought to accelerate nodule formation in some patients. On
histology pallisading histiocyles can be seen. Septal panniculitis is seen with E nodousm.
Crystals would be indicative of gout.
5) The correct answer is A.

All patients with RA should have an x-ray of the cervical spine prior to undergoing surgery to
rule out instability. X-ray should be ordered with flexion and extension views to evaluate for
atlanto-axial subluxation. Lumbar spine is not affected in RA so would not be indicated. PFTs
would not be required unless there is a history of underlying lung disease. Dopplers wouldn’t be
needed since RA patients are not necessarily thought to have increased hypercoaguable risk.

CASE 1

A 23-year-old woman is evaluated in the emergency room for fever, malaise and joint pain.
Symptoms began 5 days ago, when she had right wrist pain which resolved spontaneously. She
subsequently developed pain in the left wrist and left knee associated with mild swelling. On
exam today, she has joint swelling over the left wrist and a small left knee effusion. She is
sexually active and reports a mild vaginal discharge in the last couple of weeks.

Physical examination reveals a temperature of 101.1 degrees F, pulse 90 bpm, blood pressure
134/72. General physical exam is normal except for 4 small vesiculo-pustular erythematous
lesions noted on her left hand. Musculoskeletal exam reveals tenosynovitis and tenderness of the
left wrist, and a small left knee effusion. Other joints are normal.

Laboratory data:

White blood cell count 15,000/cu mm


Polymorphonuclear leukocyte (PMN): 75%
ESR: 63 mm/hr
CMP: normal
HIV: negative
Hepatitis serologies: negative
Arthrocentesis of the left knee is performed and synovial fluid results are as follows:
White blood cell count: 38,000
Polymorphonuclear leukocyte (PMN): 85%
Crystals and gram stain: negative

1) What is the next step for diagnosis:

A. Repeat arthrocentesis
B. Send cervical, urethral and rectal cultures
C. X-ray the left knee
D. Order ANA, RF and CCP Ab
E. Steroid injection to left knee.

2) What is the most appropriate treatment for this patient?

A. Amoxicillin PO
B. Doxycycline PO
C. Cefixime PO
D. Ceftriaxone IV
E. Ciprofloxacin IV

CASE 2

A 59-year-old Army Veteran is evaluated for swelling and pain of the right elbow. He has a long
standing history of rheumatoid arthritis and is wheelchair bound. He has had several episodes of
right olecranon bursitis over the last few years, attributed to the friction of his elbows on the arm
of his wheelchair. The last episode one year ago responded well to corticosteroid injection.
During the past few days, he has noticed redness, warmth, and pain over the elbow.

The patient’s temperature is 100.0F, pulse is 100 bpm, pulse 78, blood pressure is 125/70. On
joint exam, there is obvious swelling and tenderness and warmth of the right olecranon bursa.
There is pain with both palpation and with extension of the elbow. The patient is holding his
elbow in a flexed position. The hands show ulnar deviation at the MCP joints with synovitis.

3) What is the most appropriate next step to confirm a diagnosis?

A. X-ray of the right elbow


B. Aspiration of the olecranon bursa
C. Steroid injection of the olecranon bursa
D. Surgical referral for surgical removal for recurrent bursitis
E. Begin Cefazolin PO

Needle aspiration of the right olecranon bursa yields 10 mL of cloudy . WBC: 60,000/mm3. No
crystals seen. Gram stain reveals gram-positive cocci in clusters.

4) Which of the following antibiotics should be administered to this patient?

A. Doxycycline
B. Trimethoprin – sulphamethoxazole
C. Cefotaxime
D. Linezolid
E. Vancomycin

References and further reading:

1. Mathews CJ1, Weston VC et al. Bacterial Septic Arthritis in Adults; Lancet. 2010 Mar
6;375(9717):846-55
2. http://www.clevelandclinicmeded.com/medicalpubs/diseasemanagement/rheumatology/s
eptic-arthritis/

CASE 3
A 38 year old pregnant female is referred to you for management. She is currently 26 weeks
pregnant. She frequently visits her family in upstate New York. 8 weeks ago she developed a
swollen and painful left knee. There was no history of rash, tick bite, cardiac, respiratory or
neurologic symptoms. The left knee was aspirated and synovial fluid was slightly cloudy, with a
WBC of 18,500, and 82% PMN’s. Gram stain and cultures were found to be negative. She had a
positive Lyme titer by ELISA, and Western blot assay had shown 9 bands. (normal is less than
five.)

She was treated with ceftriaxone 2g intravenously daily for 30 days. She returns today feeling
better but still has mild arthralgias and myalgias, in addition to symptoms of fatigue. A physical
examination is unremarkable except for her pregnant state. The joint exam is largely normal,
with complete resolution of the knee effusion. Repeat Western blot assay shows seven bands
(normal <5.)

5) Which treatment option is most appropriate at this point?

A. Begin amoxicillin 2 g daily, to be continued for 3 months


B. Begin ceftriaxone 2 g IV daily for an additional 30 days
C. Begin hydroxychloroquin 400 mg daily
D. Reorder Western blot assay
E. Reassure the patient that no further treatment is needed

CASE 4

A 50 year old man is sent for evaluation for polyarthralgias. He does not complain of joint
swelling today, but relates having had two episodes of wrist and knee swelling in the last year,
which resolved after arthrocentesis and steroid injection. He has been told he may have crystal
arthropathy, but lab review shows negative crystals in both synovial fluid specimens sent in the
last year. He also has pain in the joints of his hands, ankles and feet, and morning stiffness that
lasts 15-20 minutes. Review of systems is positive for fatigue for six months and a skin rash over
his legs noticed six weeks ago. He denies fever or chills.

Past medical is unremarkable. He has had no surgeries. He denies recent travel, tobacco or
recreational drug use.

Joint exam reveals tenderness over the PIP’s, MCP’s, wrists, knees and feet bilaterally. There is
no joint swelling present. He has palpable purpura over the skin of the lower extremities.
Laboratory studies:

CBC: normal
Cr: 1.4
AST: 92 IU/L
AST: 101 IU/L
ANA: 1:40 speckled
ESR: 45mm/hr
RF: >250 IU/mL
CCP Ab: negative
C3: 50mg/dL
C4: 12 mg/dL

Further blood tests and x-rays of the joints are ordered and a dermatology consult is requested.

6) What is the most likely diagnosis:

1. Systemic lupus erythematosus


2. Reactive arthritis
3. Hepatitis C
4. Rheumatoid arthritis
5. HIV arthropathy

CASE 1

1) The correct answer is B.

This patient has disseminated gonococcal infection (DGI) with classic symptoms of
tenosynovitis, characteristic skin lesions and a migratory polyarthritis. DGI can occur as one of 2
syndromes: A triad of tenosynovitis, dermatitis and migratory polyarthralgias , or a purulent
arthritis without skin lesions. There is a degree of overlap between the two presentations.

In case of this more common presentation of the classic triad with DGI, synovial fluid cultures
may often be negative. Patients suspected of DGI should also have urethral, cervical and/or rectal
cultures submitted to increase the yield of isolating Neisseria gonorrhoea. Approximately 50%
of patients who have gonococcal arthritis have positive cultures from one of the above three
mucosal sites. The culture for N gonorrhoeae is almost always negative in skin lesions, is
positive in less than 50% of synovial fluid cultures and less than one third of blood cultures.

2) The correct answer is D.

Therapy of disseminated gonococcal infection usually consists of a third generation


cephalosporin given parenterally for the first few doses. Other choices would include IV
cefotaxime and IV ceftazadime . An update to the 2010 CDC guidelines was issued in 2012,
stating that oral cephalosporins are no longer a recommended treatment for gonococcal arthritis.
Thus PO amoxicillin or doxycycline use would be incorrect. Due to evidence of declining
cefixime susceptibility, the CDC no longer recommends cefixime (choice C) at any dose as a
first-line regimen for treatment of gonococcal infections. Quinolones are not recommended for
empiric therapy even for mild non-disseminated gonococcal infections, so D is incorrect.

All DGI cases should receive the first dose of antibiotics parenterally if treated outpatient. In a
patient who appears toxic as above, IV therapy with a third generation cephalosporin should be
initiated and continued after hospital admission. Patients should also receive 1 gram of oral
azithromycin, or doxycycline 100 mg twice a day for seven days to cover potential Chlamydia.
trachomatis co-infection. Treatment of sexual partners is also recommended.

References:

1. 1.P.A. Rice, Gonococcal arthritis (Disseminated gonococcal infection) Infect Dis Clin
North Am 19 (2005), pp. 853–861
2. http://www.cdc.gov/std/Gonorrhea

3) The correct answer is B.

Any unexplained joint or bursal fluid in a febrile patient must be aspirated to assess for infection.
Even though this patient has a prior history of chronic arthritis and similar episodes in the past,
aspiration is key to rule out infection. Having underlying joint disease and an immunosuppressed
status due to RA medications are both predisposing factors for infection in a joint.

4) The correct answer is E.

The olecranon bursa is a superficially located bursa over the olecranon surface that can become
inflamed in rheumatic conditions such as gout and rheumatoid arthritis, and also in cases of
trauma and infection. The most common organism implicated in septic bursitis is Staphylococcus
aureus, but there is an emerging role of methicillin-resistant S. Aureus (MRSA) infections even
in the community setting. The antibiotic of choice for treatment of gram positive cocci on gram
stain is IV vancomycin.

5) The correct answer is E.

This woman was appropriately treated with IV ceftriaxone for an adequate period of time. The
presence of refractory or partially treated disease should have been considered if the patient
continued to have signs of arthritis or if she developed features of Lyme carditis or neurologic
disease. However, she continues to feel well; fatigue and mild aches and pains are non-specific
and may be attributed to pregnancy. Thus she can be reassured that her Lyme disease has been
treated effectively and no further therapy is indicated.

Serologic tests may remain positive for years after Lyme disease treatment, despite the infection
having resolved. Follow up serologic testing is generally not helpful if the patient is doing well
clinically, and may confound the picture. Since antibiotic therapy does not immediately halt the
immune response, it is not unusual for follow up western blot assays to show persistence of
bands.

Follow up serologic testing may be of significance if the patient is not clinically improving, or
worsening, in which case a persistent infection may be present. However, despite this, there is no
evidence that even chronic lyme arthritis can be prevented or treated with greater than 1 month
of antibiotic course. Intravenous treatment for two weeks to one month is the CDC’s
recommended treatment duration

References and further reading:

1. Eugene D. Shapiro, M.D. Lyme Disease; N Engl J Med 2014; 370:1724-1731


2. http://www.cdc.gov/lyme

6) The correct answer is C.

The presentation of polyarthralgias, palpable purpura, elevated liver tests and a positive
rheumatoid factor (with negative CCP Ab) is suggestive of chronic Hepatitis C virus infection.
Hepatitis C infection can result in formation of circulating immune complexes which may
deposit in tissues to produce clinical manifestations of mixed essential cryoglobulinemia. These
may include arthritis, glomerulonephritis, and vasculitis.

A substantial number of patients with hepatitis C infection have musculoskeletal complaints such
as arthralgias and myalgias. Hepatitis C arthropathy may be present in 2 to 20 percent of patients
with HCV. It generally presents as a rheumatoid-like arthritis, or as an oligoarthritis.
Hepatitis C infected hepatic lymphocytes may secrete molecules with rheumatoid factor
positivity. HCV infection is associated with a positive RF in 54 to 82 percent of cases, more
commonly in patients with mixed cryoglobulinemia.

SLE can present with similar arthralgias and rash, however, an ANA titer of 1:40 is considered
negative, and the positive RF and elevated liver tests do not go along with the diagnosis.
Likewise, reactive arthritis can present with arthropathy, a similar rash, conjunctivitis and/or
urethritis. However, the patient currently is presenting with arthralgias without frank arthritis..
Rheumatoid arthritis and reactive arthritis do not typically go together with the elevated liver
tests, negative CCP Ab and low complements on blood tests. Lastly, though HIV arthropathy can
present similarly, again the constellation of laboratory tests are most suggestive of diagnosis of
Hepatitis C.

References and further reading:

1. Zeynel A. Sayiner, Uzma Haque et al. Hepatitis C Virus Infection and It’s Rheumatologic
Implications. Gastroenterology and Hepatology Volume 10 Issue 5, May 2014.
2. Rosner I1, Rozenbaum M, et al. The Case for Hepatitis C arthritis. Semin Arthritis
Rheum;2004 Jun;33(6):375-87

- See more at: http://www.rheumatology.org/Learning-Center/Educational-


Activities/Rheum2Learn/Infectious-Arthritis#sthash.z2eTSV9i.dpuf

CASE 1

A 28 year-old woman presents with fatigue, arthralgias, intermittent headaches, and easy
bruising. Basic laboratory testing is notable for a white blood cell count of 3.2, absolute
lymphocyte count of 0.8, and platelets of 15. On chart review, a complete blood count 2 years
prior was notable for a white blood cell count of 3.0, and an absolute lymphocyte count of 0.6.
Creatinine, liver function tests, albumin, urinalysis, and coagulation testing are all normal.
Autoantibody testing is notable for a positive ANA at a titer of 1:1280 with a diffuse
homogeneous staining pattern, the presence of double stranded DNA antibodies at a titer of
1:640, and hypocomplementemia. Exam is notable for malar erythema, multiple painless oral
ulcers, and ecchymoses predominantly over the extensor surfaces of her lower extremities. No
splenomegaly was noted on exam.

1) Does this patient meet classification criteria for SLE?

A. Yes
B. She has features of SLE but does not meet classification criteria
C. No

2) Which of the following etiologies should be considered in the differential diagnosis of her
thrombocytopenia?
A. Idiopathic thrombocytopenic purpura (ITP)
B. Thrombotic thrombocytopenic purpura (TTP)
C. Autoimmune hepatitis (AIH)

CASE 2

A 32 year-old male with known SLE (+ANA, +Smith, hypocomplementemia, discoid lupus
erythematosus (DLE), prior class III/V lupus nephritis, prior neuropsychiatric SLE) is admitted
with chest pain. He describes the pain as pleuritic in nature, worse with lying down, and
improved with leaning forward. He has had mild upper respiratory symptoms for 1 week, low-
grade fevers at home, and does note a non-productive cough. He has been on a stable dose of
Plaquenil 400mg daily, Mycophenolate mofetil 1000mg daily, and Prednisone 5mg daily. No
recent travel.

Vital signs in the emergency room are notable for a temperature of 100 F and heart rate of
109. His lung exam is clear to auscultation bilaterally. The remainder of his exam is notable for
hypopigmented peri-auricular lesions consistent with inactive DLE.

3) Which of the following is the least likely explanation of his symptoms?

A. Pleuropericarditis
B. Bacterial pneumonia
C. Pulmonary embolism
D. Myocardial infarction
E. All of the above need to be addressed with further evaluation

The patient has classic symptoms for pleuropericarditis (pleuritic chest pain worsened with lying
flat and relieved with leaning forward). However, particularly in patients with SLE, the
differential for cardiopulmonary processes is broad, and it is critical that multiple potential
etiologies be considered. There is a markedly increased risk of cardiovascular disease (4-fold
increased risk). Pulmonary embolism is also more common in patients with SLE (often in the
presence of antiphospholipid antibodies), as are infections, particularly in patients on
immunosuppressive therapy.

Basic labs are notable for a white blood cell count of 3.2 and otherwise normal CBC, normal
liver function tests, normal creatinine, and urinalysis without blood or protein. A chest x-ray is
done and shows a normal cardiac silhouette and very slight blunting of the costophrenic angles.
Apart from HR of 110, ECG is unremarkable. Initial troponin is negative.

4) What further workup is indicated?

A. CT angiogram of the chest


B. Echocardiogram
C. Sputum culture

5) At this point, what medications changes should be made to treat his symptoms?
A. Increase Prednisone to 60mg daily
B. Start Colchicine 0.6mg twice daily
C. Start Rituximab 375 mg/m2 weekly x 4 doses

Case 1

1) The correct answer is A.

The patient meets both the ACR classification criteria for SLE (1997 update of 1982 criteria) and
the 2012 SLICC classification criteria for SLE.

She fulfills 4 of 11 ACR classification criteria for SLE (> 4 needed to meet criteria): +ANA,
immunologic disorder (+dsDNA, hypocomplementemia), hematologic disorder
(thrombocytopenia, leukopenia on > 2 occasions, lymphopenia on > 2 occasions), malar rash,
and oral ulcers.

She also fulfills 6 of 17 SLICC classification criteria for SLE (> 4 needed with at least 1
immunologic and 1 clinical criteria met): +ANA, +dsDNA, hypocomplementemia,
leukopenia/lymphopenia, thrombocytopenia, acute cutaneous lupus (malar rash), oral ulcers.

2) The correct answers are: A and B.

Thrombocytopenia in SLE has multiple possible etiologies, including ITP, TTP, the
antiphospholipid antibody syndrome, as well as medication toxicity, viral infection, and
malignancy. Liver disease, including AIH, can lead to hypersplenism, splenic sequestration of
platelets, and consequently thrombocytopenia. However, she has normal liver function tests and
normal synthetic function on laboratory analysis, no splenomegaly on exam, and no stigmata of
chronic liver disease, making AIH extremely unlikely.

Case 2

3) The correct answer is A.

All of the above need to be addressed with further evaluation.

The patient has classic symptoms for pleuropericarditis (pleuritic chest pain worsened with lying
flat and relieved with leaning forward). However, particularly in patients with SLE, the
differential for cardiopulmonary processes is broad, and it is critical that multiple potential
etiologies be considered. There is a markedly increased risk of cardiovascular disease (4-fold
increased risk). Pulmonary embolism is also more common in patients

4) The correct answers are A and B.


Given the lack of pulmonary infiltrates on chest x-ray, bacterial pneumonia is less likely. Despite
the normal cardiac silhouette and lack of ECG changes, pericarditis remains possible and an
echocardiogram is indicated. A pulmonary embolism also still needs to be considered.

CT angiogram of the chest is negative for pulmonary embolism. Echocardiogram shows a small
pericardial effusion and is otherwise unremarkable

5) The correct answers are A and B.

The patient has classic symptoms for pericarditis and a small effusion on echocardiogram
supporting this diagnosis. Therapeutic approaches include: increasing corticosteroid dose (PO or
IV if necessary), colchicine, and consideration of NSAID therapy (to be employed with caution
in this patient with a history of lupus nephritis). In the outpatient setting, depending on his
response, a change to his baseline immunosuppressive regimen may be indicated. However,
there is not an indication for use of rituximab in this setting.

CASE 1

A 25 year old male has a 6 months history of progressive lower back pain. It began slowly
without prior trauma and it is associated with stiffness and pain, particularly in the second
portion of the night. He will awaken and need to get up and move around for a period of time
before he can return to bed. In the morning, he has prolonged morning stiffness of his back,
particularly in the buttock region. The pain will sometimes radiate down towards the posterior
knee but never below it and sometimes alternate from one side to the other. He prefers to walk
around at work rather than sit at his desk.

1) Which of the following is likely the source of his pain?

A. Spinal stenosis
B. Facet arthropathy
C. Sacroiliac inflammation
D. Sciatica
E. Infectious disciitis

CASE 2

A 30 year old female has new onset joint pain and swelling in her right index finger, left wrist,
and left ankle of 3 weeks duration. There was no antecedent trauma. She is in good health
without any other medical problems. She travelled to Central America for an extended period of
time and just returned 2 months ago. She developed an episode of what she presumed was
traveler’s diarrhea that was associated with fevers, chills, crampy abdominal pain and loose
stools. Her symptoms resolved after a week. Other companions on the same trip also developed
similar symptoms. She has tried naproxen 220mg BID with some improvement.
On exam, she is afebrile with normal vital signs. General exam is normal. Joint exam reveals
dactylitis of her right index finger, swelling in her left wrist, and medial left ankle which
is painful on inversion against resistance.

Laboratories: ESR of 45mm/hr; CRP of 30mg/L (normal <10mg/L); WBC of 13,000 cells/mm3;
hgb of 14 gm/dL and hct of 40%. Creatinine, LFTs and urinalysis are normal.

2) What is the most appropriate next step in management of this patient?

A. Switch to alternative nonsteroidal anti-inflammatory medication at high dose


B. Obtain stool cultures
C. Prescribe a course of Ciprofloxacin
D. Order HLA B27
E. Start methotrexate at 15mg po weekly

CASE 3

A 22 year old previously healthy male developed unilateral uveitis that was treated with topical
steroids successfully. 3 months later, he develops onset of left knee pain and swelling that is
associated with prolonged morning stiffness that gradually improves by noon. He also notes
chest wall pain with inspiration and bilateral heel pain. He denies any rashes, diarrhea or mouth
ulcers. There is family history of an uncle with Crohn’s disease.

Physical exam reveals normal vital signs. The general exam is normal. The chest wall is tender
at the costochondral junctions. Musculoskeletal exam reveals moderate size left knee effusion
that is warm and limited range of motion. The ankles are normal but there is point tenderness and
swelling of bilateral Achilles insertion at the retrocalcaneal region.

3) The most likely diagnosis is:

A. Rheumatoid arthritis
B. Ankylosing spondylitis
C. Undifferentiated spondyloarthritis
D. Psoriatic arthropathy
E. Sarcoidosis

4) Psoriatic arthritis has a wide range of clinical features that are clinically present in other
spondyloarthropathies. The one feature that is unique and characteristic in psoriatic
arthritis is:

A. Spondylitis
B. Arthritis mutilans
C. Dactylitis
D. Enthesopathy
E. Asymmetric arthritis
Tumor necrosis factor alpha inhibitors (TNFi) are the predominant class of biologic medications
used to treat ankylosing spondylitis, psoriatic arthritis, and inflammatory bowel disease when
more traditional medications are ineffective. The decision to use these medications includes
assessing risk factors.

5) Which of the following would be a contraindication to starting a TNFi?

A. Well controlled diabetes mellitus


B. Latent tuberculosis treated with isoniazid
C. Chronic Hepatitis B
D. Chronic renal insufficiency
E. Angina pectoris

 The correct is C.

The patient’s back symptoms are classic for inflammatory back pain (IBP). These characteristic
features include: 1) insidious onset over 3 months; 2) pain improving with activity but not with
rest; 3) nocturnal pain, particularly in the second half of the night; and 4) alternating buttock pain
suggesting SI joint involvement. Usually mechanical pain is worse with activity and improves
with rest. Spinal stenosis and sciatica have pain radiating from the low back down the legs and
extend past the knee but again, the pain is more problematic with activity. Spinal stenosis and
facet arthropathy would be highly unusual given his age. Nocturnal pain is often a worrisome
sign of infection or malignant disease within or near the spine. In this patient, given his age,
duration of symptoms and lack of systemic complaints, these two considerations are less likely
but should remain in the differential.

 The correct answer is A.

This patient has acute post-enteric reactive arthritis. By the time she presented, usually 2-6
weeks after the gastrointestinal infection, there is low likelihood that stool culture will be
positive. Antibiotics have not been proven to be effective for post-enteric reactive arthritis while
there is still some suggestion that prolonged antibiotics for post-chlamydial reactive arthritis
might benefit from prolonged antibiotics such as lymecycline, ciprofloxacin, or
azithromycin. The prevalence of HLA B27 is estimated to be anywhere from 30-50% in reactive
arthritis and not likely to predict susceptibility but perhaps predictor of more severe
disease. Since this patient is in the acute phase of reactive arthritis, a trial of different NSAIDs is
most appropriate as initial therapy and use of corticosteroids if NSAIDs are ineffective may
provide short term relief. If the patient evolves into more chronic reactive arthritis, then
sulfasalazine, methotrexate may be necessary.

 The correct answer is C.

The term undifferentiated spondyloarthritis is sometimes very useful in a patient who presents
with feature of spondyloarthritis but without associated features of psoriasis, inflammatory bowel
disease, or features of true ankylosing spondylitis. Sometimes, the joint manifestations present
prior to the onset of skin disease (psoriasis) or bowel symptoms (inflammatory bowel
disease). The unifying features of this group of diseases include uveitis, enthesitis, asymmetrical
arthritis, tendonitis, and/or inflammatory back pain. This patient’s symptoms would also fit into
the proposed ASAS classification of peripheral spondyloarthritis since most of his clinical
features are peripheral but there are patients who have component of both. Rheumatoid arthritis
can present in asymmetrical fashion but is not usually associated with uveitis or
enthesopathy. Sarcoid is associated with uveitis and arthritis /periarthritis but the enthesopathic
features of the chest wall is uncommon.

 The correct answer is B.

Arthritis mutilans is a rare but destructive form of psoriatic arthritis that is associated with
progressive loss of bone in the affected joint. This results in progressive changes and on
radiograph, the findings of “pencil cup “deformities. This subset of arthritis is very suggestive of
psoriatic arthritis even if the patient has no skin manifestations. All the other listed clinical
features above are also seen in reactive arthritis, ankylosing spondylitis, and arthropathy
associated with inflammatory bowel disease.

 The correct answer is C.

TNF alpha inhibitors have multiple potential side effects and before initiation of these drugs, risk
assessment should be carefully weighed. The most important exclusion would be the presence of
active infection or presence of latent tuberculosis. Reactivation of tuberculosis has been
observed in initial clinical trials and other fungal infections including histoplasmosis and
coccidiomycosis have also been reported. All patients should have PPD or interferon-gamma
release assay for latent TB and treatment should be initiated before start of TNFi. Other
infections include untreated hepatitis B or C. Patients with heart failure, NYHA functional class
III/IV should avoid these drugs due to potential worsening of congestive heart failure. Other
relative contraindications to these drugs include prior or current history of demyelinating disease,
history of melanoma, or pregnancy.

CASE 1

A 47 year old female patient presented with pain and stiffness in his small hand joints. He had
significant morning stiffness. In the next few months, he developed some difficulty swallowing,
and progressive difficulty sitting up in bed and getting up from a chair. On examination he had
dry cracking skin on the radial margins of his index fingers. His creatine kinase (CK) was 3000
U/L (Normal < 220 U/L).

1) The next best test for this patient is:

A. Muscle biopsy
B. Electromyogram (EMG)
C. Spirometry, transfer factor and high resolution CT scan of lungs.
D. Echocardiogram
E. Swallow evaluation

CASE 2

A 74 year old male patient was diagnosed with polymyositis. Over the last 7 years, he had
progressive dysphagia and slowly progressive muscle weakness affecting his shoulder abductors,
glutei, quadriceps, and forearm muscles. He has now become wheelchair bound.
Immunosuppressive medications including glucocorticoids, methotrexate, azathioprine,
mycophenolate, and intravenous immunoglobulin did not seem to have any significant effect on
the course of his muscle disease. On examination he was an emaciated man with significant
muscle wasting and proximal muscle weakness. He was unable to make a loop with his thumb
and his middle finger. His creatine kinase was 349 U/L (normal < 220 U/L).

The next best test is:

A. CT scan of chest, abdomen, and pelvis


B. Genetic testing for limb girdle muscular dystrophy
C. Colonoscopy
D. Muscle biopsy
E. Swallow evaluation

CASE 3

A 53 year old diabetic lady was admitted with severe generalized myalgias and swelling of her
upper arms, calf muscles and thighs. She was on metformin, glipizide, simvastatin, and enalapril.
A recent bout of sinusitis was treated with a course of clarithromycin. On examination the patient
had severe muscle tenderness, especially of the deltoids, quadriceps and calves. Muscle strength
was difficult to evaluate because of severe muscle pain with minimal movement. Her CK was
375,000 U/L (normal < 220 U/L). Serum creatinine was 1.2 mg/dL. The patient was admitted for
intravenous hydration.

The most appropriate management for this patient is:

A. Urinalysis and urine myoglobin


B. Serial CK assessments in the next 7 days
C. Autoantibody testing: Antinuclear antibody, antibodies to extractable nuclear antigens,
and myositis specific antibodies.
D. EMG
E. Muscle biopsy
F. MRI scan of thigh muscles.

CASE 4
A 32 year old female smoker presented with an erythematous scaly rash on her face and
knuckles. She had profound proximal myopathy, dysphagia, and orthopnea. There was nailfold
erythema and cuticular overgrowth. Her creatine kinase was 10,520 U/L (Normal < 220 U/L).
With a working diagnosis of dermatomyositis, she was treated with high dose prednisone and
methotrexate. As there was inadequate response in the next 2 months, methotrexate was switched
to azathioprine, and eventually monthly intravenous immunoglobulin infusions were added, but
her muscle weakness continued to worsen and creatine kinase did not go down.

The best management for this patient is:

A. ANA testing and myositis specific antibodies.


B. Chest X-ray, CA-125, pelvic ultrasound.
C. Spirometry (lying and sitting).
D. Muscle biopsy.

CASE 5

A 45 year old female was seen in clinic as she has developed significant muscle aches, cold
intolerance, fatigue, constipation, and a 10 lb. unintentional weight gain in the past 6 months.
She complained of paresthesias of her thumbs, and index and middle fingers, especially at night.
Her CK was 740 U/L (Normal < 220 U/L). On examination, her heart rate was 55/min, and BP
was 145/90. She had facial and lower extremity dry skin, coarse facial features, and some painful
proximal myopathy.

5) The test which is most likely to give the correct diagnosis is:

A. EMG
B. TSH
C. Muscle biopsy
D. Chest X-ray

 The correct answer is C.

This patient has anti-synthetase syndrome. The dry cracking skin on the radial margins of his
index fingers is described as 'mechanic's hands', which is a characteristic finding in anti-
synthetase syndrome.

Spirometry, transfer factor and high resolution CT scan of lungs will help assess the most serious
manifestation of the disease that will dictate treatment plan and prognosis – interstitial lung
disease (ILD).

A muscle biopsy will confirm the diagnosis, but the diagnosis is already apparent.
EMG may be abnormal indicating myopathy rather than neuropathy, the patient's history and
physical examination findings are already suggestive of an inflammatory myopathy and not a
neuropathy. An echocardiogram is not the logical next test as pulmonary hypertension is much
less common in anti-synthetase syndrome compared to ILD. A swallow evaluation may be
relevant later, and may already confirm what is known and expected in antisynthetase syndrome,
but will not help assess the most serious complication of the disease, that will dictate treatment
plan and prognosis, ILD.

 The correct answer is D.

The presentation is suggestive of inclusion body myositis (IBM), where an underlying


malignancy is not more common than that in the background population.a Hence CT scan of
chest, abdomen, and pelvis, and colonoscopy are not likely to be helpful. Also, for a patient
presenting so late in his life, limb girdle muscular dystrophy is distinctly unlikely. Also, the
patient has dysphagia and distal weakness in certain muscle groups, which are features
suggestive of IBM. The appropriate test will be a muscle biopsy: light microscopy is expected to
show the characteristic rimmed vacuoles, and electron microscopy will show the abnormal
filamentous bodies pathognomonic of IBM. A swallow evaluation may be relevant, and may
already confirm what is known and expected in IBM. However, it will not help differentiate IBM
from poly- or dermatomyositis.

 The correct answer is B.

The patient clearly has acute rhabdomyolysis, the likely cause of which is the addition of
clarithromycin to simvastatin. Urinalysis may show false positive test for hemoglobin (indicating
presence of myoglobin) but no RBCs, but will not help assess the response to hydration, and it
not differentiate from other causes of severe CK elevation. Serial CK assessments in the next 7
days will help assess response to hydration, and rapid normalization of CK in the next week or so
will help differentiate from inflammatory muscle diseases such as poly- or dermatomyositis,
where CK is persistently elevated over weeks to months, especially if immunosuppressive
therapy is not initiated. Autoantibody testing will not be relevant, as inflammatory muscle
diseases are not suspects here. An EMG will also not be relevant, as it will not help in the
diagnosis of rhabdomyolysis. Muscle biopsy, which may show evidence of a necrotizing
myopathy, will not be helpful and hence not indicated in rhabdomyolysis. MRI scan of thigh
muscles will also not be of any use in a patient with such widespread muscle involvement and
such severe creatine kinase elevation, which is seen in rhabdomyolysis as opposed to diabetic
myonecrosis or diabetic amyotropy, where the involvement is more focal, usually involving a
single muscle group.

 The correct answer is B.

When dealing with multi-drug refractory dermatomyositis, an underlying malignancy always


needs to be excluded. In a young female ovarian carcinoma is over-represented in patients with
dermatomyositis. Also, as she is a smoker, a chest X-ray would be a relevant initial screening
test for lung cancer.
ANA testing and myositis specific antibodies will not help further characterize the nature of
multi-drug refractory dermatomyositis. EMG will not help further characterize the nature of
refractory dermatomyositis. Also, though lying and sitting spirometry may help assess
diaphragmatic weakness, it will not help in further evaluating the cause of multi-drug refractory
dermatomyositis. Similarly, though a muscle biopsy will help confirm the diagnosis of
dermatomyositis, it is not necessary in this patient as the diagnosis of dermatomyositis is not in
question.

 The correct answer is B.

The description is consistent with a diagnosis of hypothyroidism. Serum aldolase, which is


another muscle enzyme, will not help add much to the diagnosis of hypothyroidism. EMG will
not add much to the diagnosis either, though it will confirm the diagnosis of carpal tunnel
syndrome in this patient with overt hypothyroidism. An elevated TSH will suggest the diagnosis.
If TSH is elevated, free T3 and free T4 should also be checked, which together with an elevated
TSH, will help confirm the diagnosis of hypothyroidism. A muscle biopsy may show changes
consistent with hypothyroid myopathy, but it is not required for the diagnosis. Chest X-ray will
not be relevant in the diagnosis of hypothyroidism.

CASE 1

1) Which is the next best step in the management of this patient?

A. Start cyclophosphamide
B. Right heart catheterization
C. Start Bosentan
D. Right Ankle MRI
E. Home blood pressure monitoring

2) Which of the following antibodies would be predictive of the development of


hypertensive renal crisis in this patient?

A. Anti-Centromere antibody
B. Anti Jo-1 Ab
C. Anti Mi-2 Ab
D. Anti-SCL-70 antibody
E. Anti- RNA polymerase 3 Ab

The patient returns 1 month after the intial visit because her home BP readings have been high.
She continues to have active Raynauds phenomenon. On examination P 80 BP 160/90.

3) The next best step in the management would include:


A. Continue Nifedipine, and add Enalapril
B. Continue Nifedipine and add Atenolol
C. Continue Nifedipine and add HCTZ
D. Discontinue Nifedipine and add Verapamil
E. Discontinue Nifedipine and add Clonidine

CASE 2

4) Which of the following antibodies would be most likely present in this patient?

A. Anti-Centromere antibody
B. Anti Jo-1 Ab
C. Anti Mi-2 Ab
D. Anti- RNA polymerase 3 Ab

5 years after the intitial evaluation the patient presents with increasing dyspnea on exertion for
the past 4 months. Physical exam confirms stable findings of sclerodactyly, digital pitting scars,
calcinosis over the elbows, and telangiectasias over the face and chest. Pulmonary examination is
clear to auscultation. Cardiac exam reveals a regular rate and rhythm and a loud P2 component
of S2. Pulmonary function tests reveal normal Lung volumes, and a DLCO of 40% of predicted.

5) The most likely cause for the dyspnea is?

A. Anemia
B. Congestive heart failure
C. Interstitial lung disease
D. Pulmonary HTN

CASE 1

1) The correct answer is E.

Home blood pressure is the correct answer and is a very important screening test in patients with
early diffuse scleroderma to assure diagnosis of hypertension which could lead to hypertensive
renal crisis. Cytoxan is used for treatment of active inflammatory pulmonary disease which has
not been established in this patient. Right heart catheterization would be performed if there were
additional signs and symptoms of pulmonary HTN including shortness of breath, Low DLCO on
pulmonary function tests, or increased right sided pressures on echocardiogram.The patient
denies shortness of breath, and has not yet had PFTs or echocardiogram. Bosentan is used for the
treatment of pulmonary HTN which has not been established in this patient. Ankle MRI is not
necessary, as a tendon friction rub is a clinical finding seen in patients with diffuse scleroderma,
and does not need further evaluation.

2) The correct answer is E.


Anti RNA polymerase-3 is found in patients with diffuse scleroderm, and correlates with the
development of hypertensive renal crisis and is the correct answer Anti centromere ab is see in
the clinical subset of patients with with limited cutaneous disease as in this patient, Anti Jo1 is
present in patients with antisynthetase syndrome a subset of the inflammatory myopathies Anti
Mi-2 is found in a subset of patients with dermatomyositis. Anti SCL-70 is associated with
diffuse scleroderma, and the presence of interstitial lung disease.

3) The correct answer is A.

This patient has hypertension in the setting diffuse scleroderma, and is at risk of developing renal
crisis. Continue the Nifedipine and add Enalapril is the correct answer. Addition of the ACE
inhibitor will help to prevent the development of hypertensive renal crisis, and is the drug
category of choice. Discontinue the Nifedipine and add Verapamil would not protect the patient
against hypertensive renal crisis. Continue the Nifedipine and add Atenolol would not protect the
patient against hypertensive renal crisis. Discontinue Nifedipine and add CLonidine, again would
not prevent renal crisis

CASE 2

4) The correct answer is A.

Anti centromere ab is seen in the clinical subset of patients with with limited cutaneous disease
as in this patient, and is the correct answer Anti Jo1 is present in patients with antisynthetase
syndrome a subset of the inflammatory myopathies. Anti Mi-2 is found in a subset of patients
with dermatomyositis. Anti SCL-70 is associated with diffuse scleroderma, and the presence of
interstitial lung disease. Anti RNA polymerase-3 is found in patients with diffuse scleroderma ,
and correlates with the development of hypertensive renal crisis.

5) The correct answer is D.

Pulmonary HTN is the correct answer, and is the likely diagnosis to explain dyspnea on exertion
in a patient with long standing limited scleroderma. There are no physical exam features to
suggest congestive heart failure, ie PND, LE edema, JVD, or rales Anemia can be seen in
chronic disease, and can lead to dyspnes on exertion, but would not explain the abnormal P2 or
DLcO. Interstitial lung disease would cause decreased lung volumes and a restrictive pattern on
PFTs not seen in this patient.

 Which of the following objective measures have been shown to have the greatest sensitivity
and specificity when used in combination for the diagnosis of SS?

A. Anti-Ro or La antibody, Ocular Staining Score (OSS) and Minor Salivary Gland biopsy
B. Rheumatoid Factor, Shirmer’s test, Unstimulated Whole Salivary Flow (UWS)
C. Anti-Ro or La antibody, Tear Breakup Time and Shirmer’s test
D. Anti-Ro or La antibody, Unstimulated Whole Salivary Flow (UWS) and Salivary
Scintigraphy
E. Anti- Ro or La antibody, ANA, and Shirmer’s Test

 You are taking care of the following patients with SICCA. In which patient are you most
concerned about the development of lymphoma?

A. A 25 year old woman with dry eyes and dry mouth, low titer anti-Ro and fatigue.
B. A60 year old edentulous male with a history of squamous cell carcinoma of the tongue.
He continues to smoke heavily. He has negative serologies and had radiation for his
previous tongue cancer.
C. A 70 year old edentulous woman with high titer anti-Ro antibodies, parotid enlargement,
and palpable purpura on her legs.
D. A 50 year old Black man male with hilar lymphadenopathy and a painful rash on his
shins.
E. A 35 year old woman with seropositive RA, and positive anti-Ro antibody, who is no
longer able to wear her contact lenses.

 You are following a 30 year old woman with known SS. She complains today of muscle
weakness and myalgia. Two weeks ago, she was started on hydroxychloroquine 400 mg daily.
Based on her laboratory findings below, what is the most likely finding on a kidney biopsy?

Potassium = 2.0 mEq/L (normal 3.6 to 5.1 Eq/L)


Bicarbonate = 6.0 Eq/L (normal 22 to 32 Eq/L)
Creatinine = 0.8 mg/dL (normal 0.64 to 1.27 mg/dL)
Urine pH = 8
Urinalysis: trace blood and no protein

A. Membranous Nephropathy
B. Interstitial Nephritis
C. Rapidly Progressive Glomerulonephritis
D. Vasculitis
E. Curvilinear bodies on electron microscopy

 What is the most common pulmonary manifestation of SS?


A. Non-specific Interstitial Pneumonia (NSIP)
B. Lymphocytic Interstitial Pneumonia (LIP)
C. Usual Interstitial Pneumonia (UIP)
D. Organizing Pneumonia (OP)
E. Pulmonary Arterial Hypertension (PAH)

 You are asked to see a 25 year old woman with sicca, arthritis and Raynaud’s Phenomenon.
Her serologic profile is the following: ANA 1:640, positive anti-Ro, positive anti-Smith, positive
RF and positive single stranded (ss) DNA. The presence of which auto-antibody suggests more
than a diagnosis of SS?

A. ANA
B. Anti-Ro
C. Anti-ssDNA
D. RF
E. Anti-Smith

 You are taking care of a 35 year old woman with SS and diabetes mellitus. She complains to
you about fatigue and arthralgias. She has trace synovitis in her metacarpal phalangeal joints
(MCP) and proximal interphalangeal joints (PIPs). Her sedimentation rate and C-reactive protein
are normal. Hand xrays show no erosive changes. She wishes to become pregnant in the near
future.What is the best therapy at this time?

A. Leflunomide
B. Abatacept
C. Rituximab
D. Methotrexate
E. Hydroxychloroquine

 You are asked to see a 55 year old woman with a rapidly progressive weakness in her arms
and urinary retention. Her physical examination shows upper extremity spasticity and diffusely
diminished sensation. Her past medical history is significant for SS, for which she takes
cyclosporine eye drops and daily hydroxychloroquine. She recently emigrated from Eastern
Europe. What is your next diagnostic test?
A. MRI of the brain
B. MRI of the thoracic spine
C. ANA and anti-Ro and anti-La antibodies
D. Lumbar puncture for oligoclonal bands
E. PPD

1. The correct answer is A.

The diagnosis of SS rests on objective evidence of serologic, ophthalmologic and oral


signs. A recently proposed classification criterion reviewed a range of diagnostic tests
and compared them in cases and controls. The combination that yielded the greatest
sensitivity and specificity included at least two of the following: 1) Positive serum anti-
SSA and/or anti-SSB or (positive RF or ANA titer > 1:320, 2) OSS > 3 or 3) presences of
focal lymphocytic sialadenititis with a focus score of > 1. Classification criteria are
developed to allow for the enrollment of a standardized group of patients into clinical
trials. Elements of the criteria are useful as aids in clinical practice, but may not be
applicable in every patient because of practical imitations (a labial salivary gland biopsy
may be considered too invasive or may not be feasible). Therefore, the other diagnostic
tests listed in the criterion may become valuable substitutes and clinical judgment is still
critical.

2. The correct answer is C.

Patients with SS are at a higher risk for lymphoma with reported odds ratios of up to 40.
Risk factors for the development of NHL include the length of time with the disease, low
C4, type II cryoglobulins, anemia, lymphocytopenia, hypergammaglobulinemia,
cutaneous vasculitis (palpable purpura), lymphadenopathy, splenomegaly, parotid
enlargement and peripheral neuropathy. The patient in c has long-standing disease as she
is edentulous and has glandular enlargement with palpable purpura, all risk factors for the
development of lymphoma. The patients in foil a has mild primary SS and in e has
secondary SS, but there is no information given that they have ominous findings for
lymphoma. The patient in foil b has sicca from smoking and radiation and in d has sicca
from sarcoidosis, hence neither patient is at increased risk for lymphoma associated with
SS.

3. The correct answer is B.

This patient with SS has a distal (type I) renal tubular acidosis (RTA), which is the most
common renal manifestation of SS from interstitial nephritis (IN). The pathologic
hallmark is infiltration of the interstitium with inflammatory cells (plasma cells,
lymphocytes and monocytes), tubular atrophy and fibrosis, resulting in impaired distal
acidification. Classic laboratory findings include a hyperchloremic metabolic acidosis
with a normal anion gap and hypokalemia. This patient is weak because of the low
potassium. RTA is seen early in the disease course and mostly in young patients. It does
not evolve into renal failure, but requires lifelong electrolyte replacement, as it is
refractory to immunosuppressive therapy in most cases. Although glomerular disease can
occur in SS, it is less common than IN, and would have a more active urinary sediment
and possible renal compromise. Vasculitis is an uncommon finding in SS.
Hydroxychloroquine (HCQ) therapy can cause a neuromyopathy and the classic finding
of toxicity is curvilinear bodies on electron microscopy, in any organ. The risk of HCQ
toxicity increases with time and dosage. She was recently started on HCQ and would not
have had the length of exposure needed to develop this side effect, however vigilance for
myopathy is always warranted on this therapy.

4. The correct answer is A.

SS is a systemic disease and pulmonary involvement is common with an estimated


prevalence of significant lung involvement in up to 24% of patients. Pulmonary
involvement is associated with a 4-fold increased mortality after 10 years of disease. All
of the above lung conditions can be seen in SS, and NSIP is the most common. PAH is
seen in other auto-immune diseases, such as the scleroderma spectrum and Mixed
Connective Tissue Disease (MCTD), but its occurrence in SS is rare. Pulmonary
symptoms in patients with SS should be judiciously evaluated, and when present
aggressively treated with a multidisciplinary approach.

5. The correct answer is E.

It is common to see the following auto-Abs in SS: ANA, Anti-Ro, Anti-La and RF. Anti-
ss DNA is not specific. Anti-Smith is specific for lupus and in the setting of arthritis and
Raynaud’s Phenomenon, a complete work-up for other auto-immune conditions,
including lupus should be pursued.

6. The correct answer is E.

In this patient with fatigue and inflammatory arthritis, HCQ is the best choice for several
reasons:

a. studies have shown HCQ diminishes the fatigue associated with SS,
b. HCQ is a good agent for mild inflammatory arthritis,
c. HCQ may lower serum blood sugar in diabetic patients. In fact, diabetic patients on
glucose lowering therapy should be counseled about this effect and medications
adjusted accordingly.

Methotrexate (MTX) and Leflunomide would be reasonable choices for the


inflammatory arthritis of SS; however, both are teratogenic and contraindicated in
pregnancy. Women should be off of MTX for at least 3 months prior to
attempting to conceive. Leflunomide undergoes enterohepatic recirculation and
can stay in the system for a long time (up to two years). Blood levels must be
tested and negative twice before attempting to conceive for any women who have
received leflunomide. Abatacept and Rituximab are two biologic agents which
can be used in SS; however, her manifestations are not severe enough at this time
to warrant a biologic agent.

7. The correct answer is B.

This patient has the complication of transverse myelitis from her SS. Transverse myelitis
is an inflammatory disorder of the spinal cord that presents with weakness, sensory loss
and/or bowel and bladder involvement. In SS, multiple spinal levels are contiguously
involved (3 or more levels). This is distinct from multiple sclerosis where multiple non-
contiguous levels may be involved. A lumbar puncture is indicated and oligoclonal bands
may be positive (1-2 bands) in SS, but it is much lower than the bands seen in MS (2-10
bands). The brain may be involved in SS, but that wouldnot explain her current
symptoms. In a patient with known SS, repeating her serologic tests yields no additional
information. Finally, she is at risk for TB after living in an endemic area, but she likely
has a positive PPD from BCG vaccination.

CASE 1

A 63 year-old healthy secretary presents complaining of hand and wrist pain. While the pain is
bilateral, it is more problematic in her dominant hand. It was initially intermittent but she now
notices it on a daily basis. She relates that she has stiffness in her hands in the morning lasting
for about 15 minutes. The pain is in her finger joints but spares the MCPs. In the wrist, she points
to pain at the base of her thumb rather than the wrist itself. This area is particularly painful when
she grips things with her thumb such as turning her key in her car ignition or picking up a stack
of files. She denies any joint swelling but admits that her fingers are getting "knobbier". She
relates her mother also had very crooked fingers and complained of arthritis. She denies any
other joint complaints.

On examination of the hands, she has bony enlargement with tenderness in multiple distal
interphalangeal joints (DIP) and proximal interphalangeal joints (PIP). There is also increased
pain at both the carpometacarpal joint and metacarpophalangeal joint (MCP) of the thumb. The
other metacarpophalangeal joints and the wrist are normal. There is no synovitis. The appearance
is similar to that in the attached picture.
Findings on radiograph are attached.

This patient has osteoarthritis (OA) of her hands.

1) What are the key points in her history that suggest OA?

2) What findings on physical exam support a diagnosis of osteoarthritis? Which findings help
distinguish it from an inflammatory arthropathy?

3) What features on her hand X-ray are characteristic of hand OA and was the radiograph really
necessary in making this diagnosis?

4) What laboratory studies are indicated?

5) What other therapeutic options could be considered in the future?

CASE 1

1) The most characteristic clinical complaints for OA of the hands are the short duration of
morning stiffness, and the distribution of joints involved: the distal finger joints and base of the
thumb, with sparing of the MCPs.

2) The key distinguishing feature of OA is the lack of synovitis. Bony joint enlargement is
characteristic of OA. The distribution again is more characteristic of OA than rheumatoid
arthritis (RA).

3) Radiographs were not necessary in order to make the diagnosis. Joint space narrowing with
osteophyte formation is characteristic for OA.

4) No laboratory studies are indicated for making a diagnosis of OA, unless other diagnoses are
in your differential. Baseline laboratory measurement of blood counts, and liver and kidney
function should be considered depending on what medications are instituted as therapy.

The patient is begun on daily acetaminophen and referred to occupational therapy for evaluation
both for hand exercise and adaptive equipment. She is also given a splint for the carpometacarpal
joint of the thumb of the dominant hand.

5) Other oral medications that could be tried include non-steroidal anti-inflammatory drugs
(NSAIDs) and pain medications. Some patients respond to local injections to the thumb joints.
Rarely is surgical intervention warranted.
CASE 2

A 55 year-old man with a history of RA is seen for pain in both knees with recurrent swelling of
the right knee for the past 6 months. He has been seen twice at a walk-in clinic and has had the
fluid drained. Each time the fluid has re-accumulated. He has tried increasing his oral prednisone
from 5 mg to 20 mg a day without improvement in either the pain or the swelling. He is stiff for
10 minutes in the morning and is stiff when he arises from a chair. Walking is limited because of
the pain. Because of the knee problems, he was not able to continue working and has been
unemployed for the past 6 months. He denies any other joint pain on the 20 mg of prednisone.

He relates a 15 year history of RA. He was doing well on methotrexate 15 mg weekly,


prednisone 5 mg a day and etanercept 50 mg weekly but had quit taking all but the prednisone
roughly one year prior because he thought he was doing well.

His examination is notable for difficulty arising from the chair secondary to knee pain and
stiffness. He walks with the aid of a cane. There are synovial effusions in both knees, larger on
the right. No warmth is noted. Valgus deformity is present on the right. There is pain, stiffness
and coarse crepitance with range of motion of both knees, more pronounced on the right. In the
hands there is ulnar deviation and subluxation of the MCPs with swan-neck deformities in
multiple fingers, but no synovitis. Both wrists demonstrate widening and subluxation with some
loss of flexion and extension but no warmth, swelling or pain. There is a nodule at the left
olecranon. The feet also show old deformities consistent with RA with subluxation of the
metatarsal heads but no tenderness.

1) In this patient with RA, what would be the differential for knee pain and swelling
disproportionate to the arthritis activity in the other joints?

2) What work-up would best help determine the cause of the knee problems?

3) What features of the joint fluid analysis support a diagnosis of OA?

4) What features on the knee X-ray are most characteristic of OA?

5) What evidenced-based data is there regarding joint injections, oral medications and joint
replacement for OA of the knee?

CASE 2

1) Secondary osteoarthritis is dominant in the differential. Other considerations would include


infection, crystal disease and active RA.

2) Joint aspiration of the involved knee for fluid analysis would be the most helpful.

 Sterile joint aspiration of the right knee is performed and 35 cc of yellow fluid is obtained.
Analysis of the fluid demonstrates a white blood cell (wbc) count of 750/cu mm and only 50
rbc/cu mm. Crystal examination, gram stain and culture are all negative. Radiographs of the
knee demonstrate marked narrowing of the medial joint space with moderate sclerosis of the
adjacent bony margins on the right knee and lateral joint space loss with bony sclerosis in the
left knee. A diagnosis of secondary OA is made.
 For treatment, he is given an intra-articular injection of steroids and started on a daily non-
steroidal anti-inflammatory along with a proton-pump inhibitor for gastric protection and
referred to physical therapy for quadriceps strengthening. His prednisone dose is decreased to 5
mg a day. Over the next month, he notes marginal improvement in the pain and swelling. He
continues to have a significant functional loss related to problems with the right knee and is
unable to return to work. A referral to orthopedics is made for consideration of a right knee
replacement.

3) The joint fluid is non-inflammatory, with a low total wbc. In general, wbc counts less than
3000/cu mm in synovial fluid are considered non-inflammatory.

4) Asymmetrical joint space narrowing, along with bony sclerosis are characteristic of OA. Other
characteristic findings include osteophytes.

5) Joint injections with both depot corticosteroid and hyaluronate preparations have been shown
to be beneficial. Oral therapies including acetaminophen and several NSAIDs have demonstrated
benefit. It is not possible to have a study comparing joint replacement with a sham operation but
surgical intervention in those patients who have failed all alternative treatments is generally
acknowledged as providing improved mobility and pain relief.

 You are asked to see the patient 2 for medical clearance in your hospital's emergency room.
NICE She is being admitted to orthopedics because of a right femoral stress fracture found when she
developed hip pain after she tripped and fell. She has been taking alendronate for 7 years since
you first started it as well as calcium and vitamin D. Her follow up DEXAs showed over 6%
improvement in BMD in the lumbar spine, and 4% in the total hip by the end of the 3rd year, and
the BMD had remained stable since that time last checked 2 years ago. There were no
premonitory symptoms prior to her fall. You review the x-ray which shows a stress fracture
below the trochanters. With exception of the pain, she is well with no medical contraindication
for her planned procedure. What is the most important recommendation for you to make next?

A. Change the alendronate to yearly zoledronate since the alendronate failed to prevent a hip
fracture.
B. Order a new DXA to see if there is significant change to warrant changing alendronate to
a different treatment.
C. Order x-ray of the left femur to assess whether that too is fractured
D. Order tests for secondary causes of osteoporosis since she fractured despite adhering to
alendronate therapy.

REVISE  You advise patient 2 that she is at high risk for fragility fracture and recommend alendronate
which she agrees to begin. 3 years later, you repeat a DXA, and her lumbar spine BMD has
declined 3.8% while the left total hip is unchanged. She says she has been taking her medication
faithfully once weekly on first rising in the morning on an empty stomach with a glass of water
waiting at least ½ hour before eating breakfast. Despite taking Vitamin D 1,000 IU daily, her 25-
hydroxy Vitamin D level is low 12 ng/ml (normal >30ng/ml). In addition to prescribing
additional vitamin D, what else would be most important to order?

A. Increase alendronate from 70 mg once a week, to twice a week.


B. TTG.
C. Ask patient to wait for over an hour after taking her alendronate before eating breakfast.
D. Increase patient's calcium supplement from 1000 mg daily to 2000 mg daily.

 Patient 2 responds to the alendronate 70 mg weekly you prescribed and her bone density
EASY improved 8.6% in her lumbar spine and 3.8% in the left total hip over the next 5 years of
treatment with a femoral neck T score of -1.4. You see her back and recommend she begin a
"holiday" from alendronate 2 years ago since the BMD is much improved. Her follow up DXA
off alendronate just done this year shows a stable BMD with no change from 2 years ago and you
recommend continuation of the holiday and adequate vitamin D and calcium in the diet. A
Vitamin D level is normal. The patient now has developed headaches, jaw claudication and a
temporal artery biopsy confirms Giant Cell Arteritis and prednisone is started 60 mg a day. What
would be the best next recommendation for her osteoporosis?

A. Increase calcium supplements to 2000 mg daily and Vitamin D to 50,000 IU weekly


B. Begin raloxifene daily
C. Resume alendronate 70 mg weekly
D. Make no additional treatment recommendation

 The 2nd patient returns for follow up and is concerned about your recommendation for
1200mg of daily elemental calcium and 1000 IU of daily vitamin D intake. She has heard that
calcium supplements increase the risk of heart disease. What is the evidence that calcium
supplements increase the risk of heart disease?

A. Randomized prospective and retrospective studies have shown increased calcium


deposits in coronary arteries in patients taking calcium supplements compared with
patients not taking calcium supplements.
B. Prospective and retrospective studies population studies have demonstrated increased
cardiovascular mortality in older patients taking calcium supplements.
C. Increased cardiovascular risk of taking calcium supplements has been found in patients
with hypertension taking calcium channel blockers.
D. Studies of long term use of calcium supplements have demonstrated higher incidence of
kidney stones, kidney disease, and risk of hypertension with its increased risk of
cardiovascular mortality.

REVISE  Patient 2 elects not to take the bisphosphonate you recommended for her. 6 months later she
sustains a moderately painful vertebral compression fracture at T-10. She was referred to an
orthopedist that week who recommended vertebroplasty. She sees you the next day for an
opinion about the procedure versus other management strategies both for her fracture and for her
osteoporosis. What would be the 2 most cost effective suggestions you could make regarding
her 2 concerns?
A. Have the vertebroplasty done and begin calcitonin nasal spray
B. Get an opinion about kyphoplasty rather than vertebroplasty and begin IV ibandronate
every 3 months
C. Wait another 2-4 weeks for the back pain to improve using oral analgesics and begin
alendronate 70 mg weekly
D. Wait another 2-4 weeks for the back pain to improve using oral analgesics and begin
teriparatide injections 20 micrograms subcutaneously daily.

 The correct answer is C.

Atypical fractures associated with prolonged use of bisphosphonates are rare, but when they
occur on one side, there is a high incidence of bilateral femur fractures. This patient should be
evaluated immediately for a coexisting stress fracture on the opposite side, which may be
asymptomatic (1). While the goal of osteoporosis therapy is to reduce risk of fragility fracture, a
fragility fracture does not necessarily indicate treatment failure. Treatment might be changed, but
answers A and B would not be the most important next recommendation. D would not be the
best answer. One should always consider secondary contributions to metabolic bone disease in
the setting of any fragility fracture, but again would not be the most important next
recommendation.

 The correct answer is B.

When a patient's BMD does not improve with bisphosphonate therapy, it is important to try to
determine the contributing factors. Undiagnosed secondary causes of metabolic bone disease are
not uncommon reasons. Celiac disease is relatively common in the general population and is
often unrecognized. The low vitamin D level despite the supplement suggests the patient is not
absorbing the vitamin D normally, a scenario that suggests celiac disease for which the serum
TTG would be the best initial screening test. Answer A is not correct as the alendronate
approved maximum dose is 70 mg once a week. Answer C is not the best answer. While one can
increase the absorption of alendronate by delaying breakfast after the dose, this does not address
the vitamin d problem. Likewise, increasing the calcium supplement would not address the
vitamin D problem, and higher doses of calcium would be of uncertain benefit to bone strength
and may increase risk of kidney stones.

 The correct answer is C.

This patient's risk factor profile for sustaining a fragility fracture markedly changed when she
had to begin high dose glucocorticoids for giant cell arteritis. She is now again at high risk for
losing further bone mass from the steroids and/or sustaining a fragility fracture. The American
College of Rheumatology has published guidelines to assist clinicians in the prevention and
treatment of glucocorticoid-induced osteoporosis (GIOP) referenced below. While it is important
to have adequate dietary calcium and vitamin D intake, taking additional calcium and vitamin D
above the requirements has not been shown to be helpful and may increase a patient's risk of
kidney stones. Raloxifene has not been shown to be effective in either the prevention or
treatment of GIOP. Resuming alendronate is the best answer which has been shown to be
effective in both the prevention and treatment of GIOP in randomized controlled trials.
 The correct answer is B.

Reanalysis of the Women's Health Initiative (a prospective trial designed primarily to look at the
risks/benefits of hormone replacement therapy in post-menopausal women but also looked at diet
and osteoporosis effects) and other work demonstrates that there is slight but statistically
significant increase in cardiovascular events in elderly women taking calcium supplements who
were not already on them before the study began compared with women not taking calcium
supplements controlling for then known confounding variables. Answer A is not correct as
coronary calcification has not been looked at in a randomized fashion. Answer C would not be
correct as calcium intake would not affect the mechanism of action of calcium channel blockers.
Although taking too much calcium can increase a person's risk for urinary calculi, there are no
studies showing increased incidence of hypertension in patients taking calcium supplements.

 The correct answer is C.

The back pain associated with an osteoporotic vertebral compression fracture will usually
improve greatly within several weeks without specific therapy and vertebral augmentation
procedures such as vertebroplasty or kyphoplasty are usually not necessary. The utilization and
timing of these procedures are controversial but since most patients improve without them, they
are generally reserved for patients with incapacitating pain that fails to respond to a trial of
conservative management. Most clinicians consider intranasal calcitonin to have minimal
evidence for efficacy. IV ibandronate is expensive and in prospective clinical trials has not been
shown to reduce the incidence of hip fractures and as such is not considered as effective as other
less expensive alternatives. Teriparatide is a very effective treatment for osteoporosis and in
controlled trials appears to be slightly more effective than alendronate, but is an order of
magnitude more expensive than generic alendronate and as such would not be as cost effective as
alendronate.

 A 48-year-old man presents with a 4-month history of a recurring, painful, erythematous


EASY
papular rash on the upper and lower extremities. A skin biopsy reveals leukocytoclastic vasculitis
(LCV). He is otherwise healthy and on no medications.

Laboratory studies:
Complete blood count Normal
Urinalysis Normal
C3, C4 complement Normal
ANCA Negative
Serum protein electrophoresis Normal
Cryoglobulins positive for Type II cryoglobulins
ANA Titer of 1:80

The laboratory testing which would be the most appropriate in this patient's evaluation is:

A. Anti-DNA
B. Hepatitis serologies
C. Anti-phospholipid antibodies
D. Anti-SSA, SSB antibodies

 A 76-year-old man presents with a 6-week history of fevers, fatigue, a new temporal headache,
and morning stiffness involving the shoulder and hip girdle. Two days prior to his visit, he
experienced an episode of transient diplopia.

On physical examination, he has mild scalp tenderness diffusely, with normal pulses. There is no
lymphadenopathy. Musculoskeletal examination reveals mild pain and restricted motion of his
hips and shoulders bilaterally. Neurologic examination is unremarkable.

Laboratory studies:
Hematocrit 31%
Leukocyte count 10,500/uL
Platelet count 401,000/uL
Erythrocyte sedimentation rate 68 mm/hr

The next best step in management of this patient is:

A. Refer the patient to an ophthalmologist for a dilated fundiscopic examination


B. Start prednisone 15mg daily for polymyalgia rheumatica and ask him to return in 7-10
days for a repeat examination
C. Refer the patient for a temporal artery biopsy with a return appointment after the biopsy
to discuss treatment options
D. Start prednisone 60mg daily and arrange for an urgent temporal artery biopsy

 A 47-year-old woman with a history of chronic asthma presents with numbness and tingling in
her hands and feet, with right foot "weakness". She has recently noted increased dyspnea without
wheezes, as well as PND, orthopnea and ankle swelling.

On physical exam, she has palpable purpura and papular lesions over her elbow. HEENT exam
reveals nasal polyps. On lung exam, she is noted to have bibasilar crackles without wheezes, and
an S3 gallop is noted on cardiac auscultation. Lower extremity exam reveals 1+ pitting edema.
Neurologic exam reveals decreased sensation in both feet, and reduced dorsi-flexion of right
foot.

Laboratory studies:
Hemoglobin 12.9 g/dL
Leukocyte count 12,400 /uL (46% neutrophils, 29% eosinophils, 16% lymphocytes, 9%
monocytes)
Serum creatinine 1.0 mg/dL
Creatinine kinase normal
p-ANCA negative
Urinalysis trace protein; 0-3 RBC's/hpf

Chest radiograph reveals scattered bilateral nodular infiltrates, cardiac enlargement and vascular
congestion
Echocardiogram reveals global hypokinesis with ejection fraction of 4

The most likely diagnosis is:

A. Churg-Strauss syndrome (eosinophilic granulomatosis with polyangiitis-EGPA)


B. Microscopic polyangiitis (MPA)
C. Henoch-Schonlein purpura (HSP)
D. Giant cell arteritis

 A 68-year-old man with a history of diffuse atherosclerotic vascular disease presents with
purplish discoloration of the toes of his right foot. He has noted a low-grade fever and diffuse
myalgias. Ten days ago he underwent an arterial catheterization procedure.

On physical examination he has normal blood pressure, reduced pulses in his lower extremities,
and a blotchy rash on both lower legs (livedo reticularis).

Laboratory studies:
Leukocyte count 11,500 /uL with 18% eosinophils
Erthrocyte sedimentation rate 68 mm/h
C3 decreased
C4 decreased
Rheumatoid factor negative
Anti-nuclear antibodies Titer of 1:80
Creatinine 1.8 mg/dL
Urinalysis trace protein, 0-3 RBCs/hpf
Anti-cardiolipin antibodies negative
Hepatitis serologies negative

The most likely diagnosis is:

A. Thromboangiitis obliterans (Buerger's disease)


B. Cholesterol emboli syndrome
C. Polyarteritis nodosa
D. Cryoglobulinemia

 A 27-year-old woman presents with the acute onset of a severe headache ("I thought my head
was going to explode"), which peaked in 15-20 seconds, lasted several hours, and recurred on the
2ndhospital day. There were no associated neurologic symptoms. She is otherwise healthy and
her physical exam was unremarkable.

Laboratory studies:
Complete blood count normal
Erythrocyte sedimentation rate normal
Urine drug screen positive for cocaine
Spinal fluid analysis normal
Chest X-ray normal
Non-contrast head CT normal
Contrast-enhanced head MRI normal
Cerebral angiogram suggestive of "vasculitis", with diffuse areas of vascular ectasia and stenosis
("beading")
Repeat cerebral angiogram 3 weeks later normal

The most likely diagnosis is:

A. Primary CNS vasculitis (PACNS)


B. Neurosarcoidosis
C. Susac's syndrome
D. Reversible cerebral vasoconstriction syndrome (RCVS)
E. Subarachnoid hemorrhage

 The correct answer is B.

An underlying cause for LCV is found in 60-70% of cases. Hypersensitivity vasculitis is a term
used when an offending drug is implicated. Several autoimmune conditions may be associated
with LCV, including systemic lupus and Sjogren's syndrome. Although this patient has a weakly
positive anti-nuclear antibody (ANA), he has no other symptoms to suggest lupus or Sjogren's
syndrome, and testing for antibodies to DNA, SSA and SSB is not likely to be helpful. Anti-
phospholipid antibody syndrome may cause a rash, but the most common cutaneous
manifestation is livedo reticularis, followed by digital necrosis and splinter hemorrhages. LCV
may be related to infection, and the presence of type II cryoglobulins would suggest the
possibility of hepatits C or HIV infection, and therefore hepatitis serologies would be the most
appropriate test to perform.

 The correct answer is D.

This patient likely has polymyalgia rheumatica and associated temporal arteritis. Diplopia,
although seen in only about 10 percent of cases, is quite suggestive of the diagnosis when
present. Although 15mg of prednisone daily is appropriate therapy for polymyalgia rheumatica ,
it is not an appropriate dose for the initial treatment of active temporal arteritis. The risk of
delayed therapy is visual loss, and prompt initiation of corticosteroids (1 mg/kg/d of prednisone)
is critical. Temporal artery biopsy findings do not change significantly for several weeks, and
therefore the best course of action is to promptly start treatment while awaiting the temporal
artery biopsy.

 The correct answer is A.

This patient most likely has Churg-Strauss syndrome (CSS). This form of systemic vasculitis
often occurs in patients in the setting of a long-standing history of asthma, allergic rhinitis,
sinusitis, or atopic disease. Common findings include eosinophilia, pulmonary infiltrates, skin
rash (including LCV, and occasionally a characteristic papular rash on the elbows), mononeuritis
multiplex, and constitutional symptoms. Although considered an ANCA-associated vasculitis,
only about one-half of patients with CSS will have a positive test for ANCA, usually p-ANCA
with anti-myeloperoxidase specificity. Patients who are ANCA-negative are more likely to have
eosinophilic infiltrative disease such as pneumonitis and eosinophilic cardiomyopathy, a leading
cause of mortality in CSS. The presence of neuropathy suggests the patient may be entering a
vasculitic phase of the disease, at which point asthma may paradoxically improve. Microscopic
polyangiitis (MPA) is a systemic ANCA-associated vasculitis involving the kidney and lungs,
and most patients are p-ANCA positive with anti-myeloperoxidase specificity. The absence of
both ANCA antibodies and clinical evidence of active glomerulonephritis makes this diagnosis
less likely. HSP most often occurs in children, and although it may cause palpable purpura,
abdominal pain and hematuria are frequently present. The neurologic and cardiac findings seen
in this patient would not be expected. Giant cell arteritis is a large vessel vasculitis usually
affecting individuals over the age of 50. Atlhough the aortic arch and major branch vessels may
be involved, direct cardio-pulmonary involvement and neuropathy would be unexpected
findings.

 The correct answer is B.

This patient likely has cholesterol emboli syndrome, a vasculitis mimic, with risk factors
including atherosclerosis and a recent vascular procedure. Although the physical findings,
including "purple toes" and livedo reticularis, are related to microvascular ischemia, there may
be systemic effects, possibly immune-complex mediated, resulting in elevated acute phase
reactants, eosinophilia and hypo-complementemia.

Thromboangiitis obliterans may cause digital ischemia, but is generally seen in younger male
smokers. Patients generally lack evidence of systemic inflammation , and acute phase reactants
are generally normal. Polyarteritis nodosa may cause digital ischemia and livedo reticularis, but
other common clinical features, such as abdominal pain due to mesenteric ischemia, neuropathy,
testicular pain, subcutaneous nodules, hypertension, and evidence of hepatitis B infection, are
lacking. Cryoglobulinemic vasculitis generally causes a purpuric rash, and is frequently
accompanied by neuropathy, immune complex glomerulonephritis, rheumatoid factor positivity,
and evidence of hepatitis C or other chronic viral infection.

 The correct answer is D.

This patient most likely has reversible vasoconstriction syndrome. This is a group of disorders
linked by reversible vasoconstriction of cerebral arteries. It is typically associated with severe,
acute-onset headache ("thunderclap" headache). It often occurs in defined clinical settings, such
as the post-partum state, with a migraine headache, or following ingestion of vasoactive drugs
(pseudoephedrine, ergots, cocaine, etc). Spinal fluid findings are usually normal unless there is
an associated subarachnoid hemorrhage. Cerebral angiographic findings may be
indistinguishable from primary CNS vasculitis, but should be reversible. Primary CNS vasculitis
is seen more commonly in males, and is characterized by a long prodromal period, often with
diffuse neurologic dysfunction. Spinal fluid findings are abnormal, with elevated protein and
pleocytosis, in 80-90% of patients. The combination of normal head MRI and CSF examination
makes primary CNS vasculitis unlikely. Cerebral angiography revealing ectatic and stenotic
vessels ("beading") in several sites of the cerebral circulation is suggestive, however the
relatively poor sensitivity and specificity of these findings make angiographic diagnosis
problematic. Brain or leptomeningeal biopsy may be required for diagnosis. About 5% of
patients with sarcoidosis develop CNS disease which may mimic CNS vasculitis. Common
clinical features include cranial nerve involvement, seizures, cognitive dysfunction. Contrast-
enhanced MRI generally reveals meningeal or parenchymal enhancement. Abnormal spinal fluid,
with pleocytosis and increased protein, is seen in the majority of patients. Susac's syndrome is a
microangiopathy characterized by encephalopathy, sensorineural hearing loss, branch retinal
artery occlusions, and characteristic hyperintensities in the corpus callosum on MRI.
Subarachnoid hemorrhage is unlikely with a negative head CT and spinal fluid examination.

(Answer questions 1 – 5 on a piece a paper. Find Answer Key at the bottom on the page.)

REVISE
1) Sarcoidosis can present with fever, hilar lymhphadenopathy, ankle swelling (arthritis and/or
periarthritis), and erythema nodosum. Which of the following is true:

A. This presentation is named Lofgren’s syndrome.


B. This presentation is named lupus pernio.
C. This presentation is named Heerdfort syndrome.
D. This presentation is named Koebner phenomenon.

2) AL (immunoglobulin light chain) amyloidosis is a plasma cell disorder characterized by tissue


REVISE deposition of monoclonal immunoglobulin light chains or fragments. Amyloid deposition in AL
can involve the synovium. Presentations for AL amyloidosis may include all of the following
except:

A. a chronic, progressive monoarthritis


B. subcutaneous nodules, macroglossia, & submandibular swelling
C. carpal tunnel syndrome
D. a polyarthropathy involving the shoulders, knees, wrists, and small joints of the fingers

Hemochromatois can lead to manifestations in multiple organ systems, including the MSK
system.

3) A clinically distinct form of arthropathy seen in hemochromatosis is:

A. dactylitis of the toes


B. carpal tunnel syndrome
C. arthritis of the 2nd and 3rd MCPs resembling osteoarthritis (OA)
D. sacroiliitis

Patients with autoimmune thyroid disease (AITD) includes chronic autoimmune thyroiditis
(Hashimoto’s thyroiditis) and Grave’s disease.

4) The most common serologic abnormality not directed against thyroid tissue seen in this
population of patients is:
A. anti-double stranded DNA antibody
B. ANA
C. ANCA
D. Anti-smooth muscle antibody

5) The least common musculoskeletal manifestation of diabetes is which of the following:

A. limited hand mobility (cheiropathy)


B. tendon rupture
C. adhesive capsulitis
D. Charcot joint - hand
E. flexor tendon tenosynovitis of the fingers
F. Charcot joint – foot

1) The correct answer is A.

Lofgren’s syndrome (fever, hilar lymphadenopathy, ankle swelling, and erythema nodosum) has
about 95% specificity for sarcoidosis. It is more common in Europeans than African-
Americans. Lupus pernio is a distinctive rash of sarcoidosis, often seen about the nares, that is
violaceous and may take the form of plaques or nodules. Heerdfordt syndrome, also called
uveoparotid fever, consists of fever, uveitis, and parotitis with or without CN VII
involvement. It is a relatively rare presentation for sarcoidosis. Koebner phenomenon can occur
in sarcoidosis when sarcoid skin lesions develop on old scars or tattoos.

2) The correct answer is A.

AL amyloidosis can present with soft tissue involvement. This may take the form of carpal
tunnel syndrome, a polyarthropathy, subcutaneous nodules that might resemble rheumatoid
nodules, submandibular gland involvement, and macroglossia. A monoarthritis would be very
extremely unusual.

3) The correct answer is B.

There is a predilection for the finger MCP joints, variably accompanied by chondrocalcinosis, in
the arthropathy of hemochromatosis. It is a chronic and progressive arthropathy with a low-
grade inflammation.

4) The correct answer is B.

Physicians should appreciate that a positive ANA is seen in almost 50% of patients with
autoimmune thyroid disease (AITD) – a much higher prevalence than is seen in the general
population and these patients do not usually have a classifiable rheumatic disorder.

5) The correct answer is D.


Diabetes can have a number of MSK manifestations. Limited hand mobility, also called diabetic
cheiropathy, results from increased stiffness and thickening of the finger tendons. Adhesive
capsulitis can affect 30% of diabetics and flexor tenosynovitis in the hands can affect 10-20% of
patients. Charcot joint, also known as neuropathic arthritis, usually affects the foot, and is
characterized by fracture, dislocation, and subluxation of the affected joint in the presence of a
significant sensory deficit. A Charcot joint of the hand is distinctly unusual.

CASE

An otherwise healthy 45 year old female presents to the primary care office with complaints of
pain in the joints of her hands. She notes that her wedding ring won't fit on her finger any more.
She tells you that when she is doing her job as a butcher her hand pain worsens. She has also
noted times when her hands feel numb, especially in the morning. She thinks that her hands are
starting to look like those of her grandmother's. She tells you that she was reading about her
problems on the internet and wonders if she may have gout. She denies any other specific
complaints, including fever, rash, diarrhea, and dysuria.

On physical examination, she has bony enlargement of her proximal interphalangeal (PIP) and
distal interphalangeal (DIP) joints with mild tenderness and no warmth at the DIP joints. She is
able to close her fist fully and her hand grip is normal.

1) Which of the following is the most appropriate next diagnostic step for this patient?

A. MRI of the hand.


B. Ultrasound-guided aspiration of the DIP joint.
C. Blood testing to include ANA, rheumatoid factor, and sedimentation rate.
D. No diagnostic testing.

2) The bony prominences at the patient's DIP joints are also known as:

A. Tophi.
B. Boutonnière deformities.
C. Rheumatoid nodules.
D. Heberden's nodes.

3) If the patient progressed to develop pain at the base of the thumb (first carpometacarpal
joint) along with limited abduction of the thumb, the next most appropriate step would be:

A. Referral to an occupational therapist for splinting of the thumb.


B. Wrist cock-up splints for carpal tunnel syndrome.
C. Intra-articular injection of hyaluronic acid derivatives in the first carpometacarpal joint.
D. Weekly low-dose oral methotrexate.
1) The correct answer is D.

According to the American College of Rheumatology's clinical criteria, hand osteoarthritis can
be diagnosed if hand pain is accompanied by at least three of the following features:

 Hard tissue enlargement of 2 or more of 10 selected joints (second and third PIP and DIP
as well as the first CMC joint of both hands)
 Hard tissue enlargement of 2 or more DIP joints
 Fewer than 3 swollen MCP joints
 Deformity of at least 1 of 10 selected joints

This classification method has a sensitivity of 94% and a specificity of 87%. Additional
diagnostic testing is not warranted because it has no impact on disease management. MRI can be
used to detect soft tissue abnormalities such as tendonitis or early appearance of bony erosions
but is not necessary to diagnose osteoarthritis.

Ultrasound-guided aspiration is useful to accurately obtain synovial fluid from small joints or in
cases where the anatomy is obscure. In this patient, there is no evidence of effusion and so no
need for attempted joint aspiration.

Serologic testing can be useful in the diagnosis of inflammatory arthritis such as rheumatoid
arthritis, but in this patient with pain in the distribution of hand osteoarthritis and no alarm
symptoms of inflammatory arthritis, is unnecessary.

Source: Altman R, Alarcón G, Appelrouth D, Bloch D, Borenstein D, Brandt K, et al. The


American College of Rheumatology criteria for the classification and reporting of osteoarthritis
of the hand. Arthritis Rheum 1990; 33:1601-10.

2) The correct answer is D.

These bony prominences at the DIP joints are commonly found on examination of the hands of
patients with osteoarthritis. While Heberden's nodes may initially be associated with morning
stiffness, erythema, and soft tissue swelling, this eventually subsides, leaving the patient with
bony enlargement. Similar bony prominences at the PIP joints are known as Bouchard's nodes.

Tophi are deposits of monosodium urate crystals seen in people with high levels of serum uric
acid, and are pathognomonic for gout. They can be seen at the joints but also in the soft tissue.

Boutonnière deformity refers to flexion of the PIP joint accompanied by hyperextension of the
DIP joint. This can be seen in rheumatoid arthritis as well as other disease processes affecting the
hands, such as tendon laceration, fracture, or dislocation.

Rheumatoid nodules are subcutaneous soft tissue swellings typically found over the elbows and
fingers in patients with rheumatoid arthritis. Less commonly they can found in internal organs
such as the lungs.
3) The correct answer is A.

Referral to an occupational therapist for splinting of the thumb. There are relatively few RCTs
regarding interventions for hand OA in the literature. The American College of Rheumatology's
2012 recommendations for the use of non-pharmacologic and pharmacologic therapies in hand
osteoarthritis suggests that splints for the first carpometacarpal (CMC) joint may alleviate some
pain in patients with OA at the base of the thumb.

Wrist cock-up splints are typically used for carpal tunnel syndrome to relieve pressure on the
median nerve as it passes through the carpal tunnel, which would not benefit thumb
osteoarthritis.

There is limited evidence on intra-articular hyaluronic acid derivative injections in the first
carpometacarpal joint and their use is not recommended.

Weekly low-dose oral methotrexate is used for the treatment of rheumatoid arthritis rather than
osteoarthritis.

CASE 1

A 70 year old man presents to clinic for progressive shoulder pain. He complains of mild
bilateral shoulder discomfort for the past ‘few years’, but has had progressive worsening at the
right shoulder for three months. There is no history of recent or prior trauma to this area. He has
chronic lower back pain from degenerative arthritis, but outside of this area and the involved
shoulders there are no new sites of joint nor muscle pain. He describes both pain and stiffness.
He is uncertain if activity improves his symptoms as he has avoided overhead activity secondary
to pain, and he is also not engaged in regular exercise due to his chronic back symptoms. He
localizes the pain to the upper and outer portion of the right shoulder in the deltoid region.

A review of systems is otherwise negative.

Joint examination reveals normal range of motion without swelling nor tenderness at the fingers,
wrists and elbows bilaterally. The right and left shoulders are without obvious effusion. There is
normal passive range of motion with external rotation bilaterally, with slightly limited range of
motion with internal rotation on the right. Additional shoulder examinations were performed in
an effort to localize the source of pain.

Joint examination of the lower extremities was normal other than mild limitation in range of
motion at the hips and discomfort at the lumbar spine through active flexion and extension.

The rest of the physical examination is normal.

REVISE 1) Which of the following clinical tests is not designed to identify pathology in the subacromial
space?

A. Hawkins test
B. Yergason test
C. Neer sign
D. Painful arc

Provocation testing of the right shoulder revealed discomfort with Hawkins test and Neer sign, as
well as during painful arc testing. Drop arm testing on the right was normal. Formal strength
testing of the right shoulder revealed 5/5 strength in resisted abduction, with breakaway
weakness secondary to pain. Strength was normal in internal and external rotation on the right,
with discomfort noted by the patient during the examination.

Subacromial bursitis is suspected based upon the clinical and examination findings. Non-
steroidal anti-inflammatory medications are recommended, along with modification of activities
and rest.

REVISE 2) Strengthening which of the muscles below would be most likely to provide benefit for this
condition?

A. Deltoid muscle
B. Trapezius muscle
C. Rotator cuff muscles
D. Biceps brachialis

The patient returns for follow up three months later, but unfortunately has not improved despite
good compliance with home exercises and anti-inflammatory medications. He continues to have
discomfort with Hawkins, Neer, and painful arc testing, with normal strength in abduction as
well as internal and external rotation. Internal rotation at the right shoulder continues to be
moderately limited. A decision is made to perform a subacromial steroid injection, along with a
referral to physical therapy. Upon return to clinic 4 weeks later he reports only mild, temporary
improvement in his symptoms, which after 3 weeks returned to his baseline level of pain. A plain
radiograph if the shoulder is obtained.

3) What finding on X-ray likely account for the patients continued symptoms?
A. Fracture of humeral head
B. Calcific tendinitis of the supraspinatus ligament
C. Rotator cuff tear
D. Acromial osteophyte

CASE 2

A 38 year old accountant presents with symptoms of worsening anxiety and restlessness for two
months. She has noticed perspiration even while at rest, with the onset coinciding with her
increase in anxiety. She has a history of IBS, and believes the frequency of her loose stools has
increased during this time period as well. She does not recall recent sick contacts and has not
noticed fevers nor chills. She has not traveled outside of the city for the past several years.

PMH: IBS; Fibromyalgia; Mild anxiety disorder

Medications: Dicyclomine 20mg three times daily; Amitriptyline 20mg at night; venlafaxine
150mg twice daily; tramadol 100mg three times daily as needed

Exam: VS: Temperature 38.1, HR 100, RR 26, BP 135/80

Musculoskeletal exam: joint examination is normal without synovitis nor deformity.


Neuro exam: she does not exhibit involuntary movement, although a slight resting tremor at the
bilateral hands is noted on exam. Muscular strength is normal. Deep tendon reflexes: patellar
reflex 3+ bilaterally, Achilles 3+ bilaterally, biceps 4+ bilaterally, brachioradialis 3+ bilaterally.

The rest of her cardiovascular, pulmonary, gastrointestinal, and skin exam are normal.

Lab orders are placed: CBC, comprehensive metabolic panel, urinalysis, TSH, urine toxicology
screen (to identify ingestion of illicits such as cocaine or amphetamines).

4) In conjunction with this laboratory testing, which of the options below best describes the
additional work-up/management that is needed at this visit?

A. Brain MRI with and without contrast


B. EMG/NCV of peripheral nerves
C. Stop current medications
D. CSF cultures

CASE 3

A 55 year old female presents with joint and muscle pain for the past 5 years that has been
progressively worse. She describes diffuse pain in her hands, hips, and shoulders. There has been
no obvious joint swelling, and the discomfort is described as being generalized to the entire
region of her hip and shoulder girdle, as well as the entire hand. Pain is the predominant
symptom, with report of stiffness as well. She is unable to engage in exercise or other significant
physical activity because of her symptoms.

Review of systems: periodic areas of numbness in limbs, present for past year; progressive,
‘disabling’ fatigue; poor sleep attributed to pain and anxiety; poorly controlled depression (not
on therapy secondary to intolerance and/or weight gain on agents in past); memory issues,
complaint of ‘mental slowing’

Prior lab testing that is brought with her to the appointment:

CBC, CMP & UA normal; ESR, CRP normal: 1 month prior

TSH and Vitamin D level normal: 3 months prior

ANA 1:80, with negative ‘reflex panel’; ESR, CRP normal; TSH, Vitamin B12, folate all
normal: 1 year prior

ESR, CRP normal: 2 years prior

On joint exam there is no synovitis, with full range of motion of all extremities. Muscle strength
testing of the upper and lower extremities is normal. On palpation, she describes pain upon light
pressure over the superior aspect of the posterior cervical muscles, trapezius, and deltoid area
bilaterally. On examination of the lower body she displays similar discomfort to palpation of the
gluteal and greater trochanteric region, as well as the anterior quadriceps muscle bilaterally.
Palpation of regions at the anterior cervical spine and chest, as well as regions in the distal arms
and legs bilaterally, did not elicit painful response. The rest of her physical examination is
normal.

5) Based upon the clinical scenario above, which of the following statements best describes
the suspected diagnosis?

1. Fibromyalgia is ruled out given < 11/18 tender points on examination


2. Additional diagnostic and imaging studies should be performed to rule out an
inflammatory rheumatic disease
3. A clinical diagnosis of fibromyalgia can be made
4. A clinical diagnosis of fibromyalgia can be made if brain MRI and CSF testing reveals
evidence of a pain processing disorder

CASE 4

A 56 year old man comes in for evaluation of right hand pain, stiffness, and swelling for two
months. The pain is located diffusely across the hand from his finger tips to his wrist. He
describes this as 9/10 in intensity, sharp in quality. There is a mild dull ache to the forearm and
upper arm, without swelling. There is no prior history of joint swelling or stiffness. He denies
trauma, as well as recent sick contacts nor recent travel. Outside of his right hand and arm
symptoms, he denies other joint issues.

Past medical history: Hypertension; carpal tunnel syndrome, with surgical release 3 months ago

Review of systems: negative outside of current symptoms

On exam there are no signs of infection nor poor wound healing at the post-surgical site. The
right hand is slightly cool to touch throughout, tender to touch, with diffuse soft tissue swelling.
Examination of the skin reveals brisk capillary refill. There are no skin ulcerations or signs of
infarct at the fingertips. Joint examination outside of the right hand is normal. The rest of the
physical exam is normal.
6) Which of the following diagnostic studies is most likely to help choose the appropriate
medical therapy?

A. Bone scan
B. Nailfold capillaroscopy
C. Rheumatoid factor and anti-CCP antibody testing
D. ANA antibody testing

1) The correct answer is C.

In the evaluation of shoulder pain, it is important to remember that the majority of etiologies
arise from periarticular structures rather than intracapsular disease. Pain that is worsened with
active (rather than passive) range of motion testing is suggestive of a muscle or tendon source of
pain. Having the patient identify the location of maximal pain, along with provocation testing,
can often localize the site of pathology.

Yergason testing is performed in the evaluation of suspected biceps tendonitis. It is not designed
to identify pathology arising from the subacromial space (supraspinatus tendinitis or tear,
subacromial bursitis, impingement syndrome). Patients with biceps tendonitis often localize
discomfort to the anterior aspect of the shoulder, rather than the lateral aspect as in this case.
Biceps tendonitis should not cause significant pain with overhead activity.
Hawkins testing, Neer sign, and painful arc testing all evaluate for pathology in the subacromial
space. The description of pain with overhead activity raises suspicion of a disorder in this region
and should prompt further evaluation with these tests.

Hawkins testing and Neer sign are very sensitive tests to localize injury to the subacroimial space
(roughly 80%), but specificity can be poor (30%). As such, a negative test is helpful in ruling out
disease in this area.

Painful arc testing is performed by the examiner stabilizing the shoulder with one hand while
passively abducting the arm. A positive report of pain between 60 to 120 degrees is consistent
with disease in the subacromial space (specificity 80%). Pain during the arc beyond 120 degrees
may be a sign of acromioclavicular disease.

2) The correction answer C.

Subacromial bursitis and rotator cuff tendinitis (often supraspinatus tendinitis when referring to
conditions localized to the subacromial space) often result from impingement of those structures
between the undersurface of the acromion and the humeral head. This occurs when the interval
between the acromion and humeral head is narrowed.

The deltoid is the major abductor of the arm, and also serves to pull the humeral head superiorly.
The rotator cuff muscles are involved with internal and external rotation at the shoulder, as well
as shoulder abduction. In addition, they exert a downward pull on the humeral head, acting as
stabilizer to keep the humeral head down in the glenoid fossa as the arm is abducted. Exercises to
strengthen the rotator cuff will help ‘open’ the space between the acromion and humeral head in
individuals with weak rotator cuff muscles who have an imbalance between the deltoid and
rotator cuff muscles.

The trapezius muscle elevates the scapula, while the biceps radialis serves to flex the arm at the
elbow and supinate the arm. Strengthening of these muscles would not serve to relieve
impingement in the subacromial space.

3) The correct answer is D.

A humeral head fracture can cause significant shoulder pain, but the absence of trauma and lack
of evidence for an occult fracture on plain films rule out this etiology.

Calcific tendinitis and chronic rotator cuff tear (with subsequent weakening of the rotator cuff
and development of the so-called ‘high riding shoulder’) are both conditions which can lead to
recurrent or prolonged symptoms stemming from subacromial bursitis. The radiograph from this
patient does not show evidence of either of these conditions (1, 2).

The radiograph shows evidence of a large osteophyte on the undersurface of the acromion. This
decreases the space which the supraspinatus and subacromial bursae occupy. Despite adequate
exercises to strengthen the rotator cuff, the interval may still be narrowed secondary to the
presence of a large bone spur; referral to orthopedics should be discussed with the patient.
4) The correct answer is C.

Serotonin-syndrome results from excessive serotonergic activity in the central nervous system,
and commonly is the result of medication use/medication interaction. Clinicians who manage
depression and other mood disorders, as well as fibromyalgia, need to be aware of this syndrome
given that many of the medications used to treat these conditions can raise levels of serotonin.
Selective serotonin reuptake inhibitors (SSRIs) are well known to increase serotonin levels, but
many other medications can be culprits as well, including trazodone, amitriptyline, and tramadol.
Concurrent use of multiple medications may elevate the risk for development of serotonin
syndrome.

The syndrome classically consists of changes in mental status (anxiety, delirium, restlessness),
autonomic hyperactivity (increased heart rate, increased temperature, sweating, elevation in
blood pressure, diarrhea), and neuromuscular hyperactivity (tremor, increased deep tendon
reflexes, myoclonus and muscle rigidity). The syndrome represents a clinical spectrum spanning
from milder to more severe presentations, with more severe forms thought to be reflective of the
degree of serotonergic activity. Treatment consists of stoppage of the medications that are
increasing serotonin levels. Whether all such medications need to be stopped simultaneously and
immediately depends on the severity of the presentation. Additional intervention such as
temporary use of benzodiazepines has been described, and more severe cases may require
hospital admission for monitoring and additional management.

Brain MRI and EMG/NCV are not immediately necessary in this case. Hyperreflexia is present
diffusely, with no localizing disease, which is reassuring that a focal lesion is not driving current
symptomatology. As such a search for a more generalized process should begin first. CSF culture
is not likely necessary at this time given lack of signs of infection (no symptoms of headache,
neck stiffness, or mention on exam of neck stiffness).

Close monitoring during withdrawal of medications will be necessary; if symptoms persist then
consideration should be given to these and other diagnostic tests.

5) The correct answer is C.

While prior classification criteria for fibromyalgia included reliance upon specific tender points
on exam, more recent understanding of the syndrome places an emphasis on widespread pain and
presence of associated symptoms such as sleep disturbance, mood disorder, bowel and bladder
irritability, and multiple somatic complaints. As such, the failure to reach > 11/18 tender points is
not thought to rule out the diagnosis of fibromyalgia.

Additional testing for an inflammatory rheumatic disease is not likely necessary based upon the
clinical symptoms and work-up to date. The lack of joint swelling, muscle weakness or other
objective exam findings, coupled with lack of end-organ disease by lab testing only 1 month
prior, suggest a diagnosis other than a systemic inflammatory process. In general, the absence of
clinical, radiographic and laboratory evidence of an inflammatory process after > 2 years of
symptoms suggests a non-inflammatory etiology.
Research has identified objective abnormalities on functional brain MRI and in CSF
neurotransmitter levels in patients with fibromyalgia. While providing valuable insight in to the
pathophysiology of pain processing, they are considered a research tool and are not used in the
clinical evaluation of suspected fibromyalgia.

6) The correct answer is A.

The clinical history and appearance on examination of the right hand are consistent with a
diagnosis of complex regional pain syndrome (CRPS). This syndrome may follow a precipitating
event, and an association with surgical procedures such as carpal tunnel release has been seen. A
bone scan can be helpful in CRPS to help confirm the diagnosis, and it may help in predicting
response to prednisone as well. Early in disease, diffuse uptake is seen across the finger and wrist
joints. Later in disease, the sensitivity of bone scan drops significantly. As such, a negative study
does not rule out disease, but may suggest a low likelihood of response to prednisone.

Nailfold capillaroscopy can be a useful tool to aid in the diagnosis of systemic sclerosis and other
rheumatic conditions. Early onset systemic sclerosis may present with bilateral finger puffiness.
The unilateral swelling in this case would be highly atypical of systemic sclerosis. In addition,
the brisk capillary refill is also not consistent with the vasculopathy seen in scleroderma, and is
suggestive of a hyperemic state.

Rheumatoid factor, anti-CCP, and ANA testing are unlikely to help direct medical therapy based
upon lack of clinical or exam evidence of rheumatoid arthritis or another rheumatic condition at
this time.

REVISE 1. Which patient with JIA has the highest risk of developing asymptomatic uveitis?

A. 9 year old boy with enthesitis and hip arthritis


B. 5 year old girl with systemic JIA
C. 3 year old girl with oligoarticular JIA
D. 10 year old girl with RF positive polyarticular JIA
E. 6 year old boy with RF negative polyarticular JIA

A 5 year-old boy develops a daily high fever for two weeks with a salmon-colored rash that
REVISE comes and goes. Within 2 months of presentation, arthritis of the wrists, ankles, knees and
hips develop. NSAIDs provide only minimal benefit. One month later, he develops a change in
mental status, hepatitis, purpura, and pan-cytopenia. The most likely diagnosis is:

A. Sepsis
B. Methotrexate induced hepatotoxicity
C. Macrophage activation syndrome
D. Megaloblastic anemia
E. Non-Hodgkins lymphoma
REVISE 3. A 16 year-old girl develops myalgia and rash on her knuckles. On examination, you find
papules over the PIPs and MCPs with slight violaceous discoloration and scaling.
Proximal muscle weakness of the shoulder and pelvic girdles is present. Which of the
following is NOT a well-recognized complication of this disease?

A. Gastric perforation
B. Calcinosis
C. Lipodystrophy
D. Digital infarcts
E. Renal failure

4. A 15 month-old boy develops high fever for 1 week associated with lymphadenopathy,
non-purulent conjunctivitis, edema of the hands, strawberry tongue, and a rash. Which
of these interventions will help prevent the major long-term morbidity associated with
this disease?

A. Penicillin prophylaxis
B. Regular ophthalmologic screening
C. IVIG infusion
D. Vaccination against influenza
E. Methotrexate

REVISE 5. A four year-old girl develops a rash and painful ankles. On exam you find palpable
purpura on the lower extremities and bilateral swollen, painful ankles with limitation
of motion, mostly because of pain. Late morbidity in this disease is largely due to
involvement of what organ system?

A. Renal
B. GI
C. Neurologic
D. Cutaneous
E. Cardiac

Answer Key

1. The correct answer is C.

The risk factors for development of asymptomatic uveitis associated with JIA include
ANA positivity, young age, female sex, and being early in the disease course. Systemic
JIA and RF-positive disease are not associated with uveitis. Enthesitis-related arthritis is
associated with acute, as opposed to asymptomatic, uveitis.
2. The correct answer is C.

The initial high presentation of daily high fevers, evanescent rash, and arthritis is strongly
suggestive of systemic JIA. A life-threatening associated condition is macrophage
activation syndrome. Sepsis (Answer A) could explain the constellation of features.
However this patient is not on any immunosuppressives that would put him at higher risk
for this. You are not told that this patient is on methotrexate (Answer B). Megalobastic
anemia (Answer D) can cause pancytopenia and mental status changes. It should not,
however, cause purpura and hepatitis, and in the absence of a known cause, it would be
unlikely that this patient has megaloblastic anemia. There is no association of non-
Hodgkin’s lymphoma (answer E) with systemic JIA. While lymphoma may cause fevers
and arthritis, it usually affects a few joints rather than the many joints described here, and
is not associated with an evanescent rash.

3. The correct answer is E.

Renal disease (answer E) is not commonly associated with JDM. The other answers are
well-known complications.

4. The correct answer is C.

The combination of high fever with lymphadenopathy, non-purulent conjunctivitis,


extremity edema, mucous membrane changes, and rash is strongly suggestive of
Kawasaki Disease. IVIG (Answer C) has been proven to be highly effective at preventing
coronary aneurysms. Penicillin prophylaxis (Answer A)is used for prophylaxis of cardiac
disease in acute rheumatic fever, which this patient does not have. Regular
ophthalmologic screening (Answer B) is recommended to evaluate for silent uveitis in
patients with JIA. Influenza vaccine (choice D) is recommended for patients on
immunosuppression, which you are not told this patient is on. Methotrexate (Answer D)
is used to treat and prevent long-term joint damage associated with JIA, which this
patient does not have.

5. The correct answer is A.

Long-term morbidity in Henoch-Schonlein Purpura is related to renal disease (Answer


A), which can cause nephrotic syndrome, acute renal failure, or chronic renal failure.
While GI (Answer B) and cutaneous (Answer D) manifestations can cause short-term
discomfort, they are rarely associated with long-term sequelae. Neurologic (Answer C)
and cardiac (Answer E) morbidity can occur, but they are much less common than renal
disease.

You might also like